PICU Boards Flashcards
A 3-year-old male child is found playing with an open bottle of his grandfather’s pain medications (opioids). He is sleepy, so his mother rushes him to the ED, which is not far from her house. Upon arrival to the ED, he has perioral cyanosis and the arterial saturations are reading 88% on room air. The rest of his vital signs are within normal range except for a respiratory rate of 12 breaths per minute. His lung examination is normal and his neurological examination is significant for lethargy. Which one of the following options best describes his alveolar arterial oxygenation (A-a) gradient and arterial partial pressure of CO2 (PaCO2) levels?
A - Normal A-a gradient and increased PaCO2
B - Increased A-a gradient and increased PaCO2
C - Normal A-a gradient and decreased PaCO2
D - Increased A-a gradient and normal PaCO2
E - Normal A-a gradient and normal PaCO2
A - Normal A-a gradient and increased PaCO2
The patient in this scenario has ingested pain medication which has depressed his respiratory drive. Opioids are known to depress respiratory drive, leading to hypoventilation. Hypoventilation leads to an increase in PaCO2 (making options B, C, D, and E all incorrect). His SaO2 is low because of arterial hypoxemia. Arterial hypoxemia indicates limitation of pulmonary gas exchange. There are 4 reasons for limitations of pulmonary gas exchange: (1) hypoventilation; (2) shunts; (3) diffusion abnormalities; (4) ventilation/perfusion mismatch. The A-a gradient is increased in all conditions except for hypoventilation (making options B and D incorrect). Hence, this child will have a normal A-a gradient and increased PaCO2 (making option A the correct response option).
The internal diameter of the trachea of an infant is 8 mm. As a result of airway edema, consequent to viral infection (croup), the diameter is now reduced to 4 mm. Which one of the following options will be its effect on the resistance to airflow in the airways?
A - Remain the same
B - Double
C - Increase 4 times
D - Increase by 8 times
E - Increase by 16 times
E - Increase by 16 times
The relation between flow of air through a tube and radius of the tube was described by French physician Poiseuille. In straight circular tubes, the volume flow rate is given by the following equation: V = Pπr4 / 8 ηL. Where P is driving pressure, r = radius, η = viscosity, and L = length. Because flow resistance is driving pressure divided by flow, the equation can be rearranged as the following: R= 8 ηL / πr4. Tube radius is of critical importance, if the radius is halved, the resistance increases 16 fold. However, doubling the length only doubles the resistance
A 9-year-old girl presents with acute decompensated heart failure. She has a history of adequately treated end-stage heart failure and is currently hemodynamically stable. Which one of the following treatments should be administered following admittance to the cardiac intensive care unit?
A - Dopamine
B - Epinephrine
C - Positive pressure ventilation
D - Milrinone
E - Sedation
D - Milrinone
This question describes the most appropriate treatment for a child in end-stage heart failure with acute decompensated heart failure who is currently hemodynamically stable. Options A and B are incorrect because dopamine, dobutamine, and epinephrine are typically administered to children with acute decompensated heart failure who are not hemodynamically stable. Option C is incorrect because the patient does not require positive pressure ventilation at this time because she is hemodynamically stable. Option D is correct as inotropic treatment with either milrinone or dobutamine should be initiated in patients with acute decompensated heart failure that are hemodynamically stable. Option E is incorrect because sedation should be administered in cases of left ventricular failure and pulmonary hypertension.
A 15-year-old male (weight 40 Kg) admitted with hyponatremic dehydration with serum sodium of 120 mEq/liter. If you want to correct his sodium deficit using a 3% saline infusion, what would be the maximum rate of infusion of the 3% solution (assuming, total body water 60% of body weight, target serum sodium 135 mEq/lt and target maximum increase in serum sodium 0.5 mEq/lt/hour)?
a. 20 ml/hour
b. 23 ml/hour
c. 26 ml/hour
d. 29 ml/hour
e. 32 ml/hour
b. 23 ml/hour
When corrective therapy requires the inclusion of isotonic saline or hypertonic saline, the replacement therapy can be guided by the calculated sodium deficit. This is determined as follows: Sodium deficit (mEq) = normal TBW × (135 - current PNa). Normal TBW (in lt) is 60% of the lean body weight (in kg) in males and 50% of the lean body weight in women. Thus, for a 40 kg male with plasma sodium of 120, the sodium deficit would be 0.6×40 × (135-120)= 360 mEq. Due to the fact that 3% sodium chloride contains 513 mEq of sodium/lt, the volume of hypertonic saline needed to correct sodium deficit of 360 mEq will be 0.700 lt. Using a maximum rate of rise of 0.5 mEq/lt/hr for plasma sodium, the sodium concentration deficit of 15 mEq/lt should be corrected over at least 30 hours. Thus, the maximum rate of hypertonic fluid administration will be 700/30 = 23 cc/hr.
Which one of the following options best explains pulsus paradoxus in severe asthma?
A - Inhaled ß2 agonist causes diastolic hypotension leading to myocardial ischemia, as cardiac blood supply occurs during diastole
B - Children with asthma are dehydrated on presentation—this leads to pulsus paradoxus
C - Children with severe asthma have pericardial effusion due to inflammation that leads to pulsus paradoxus
D - Frequent asthma exacerbations lead to chronic obstructive pulmonary disease (COPD)—a COPD exacerbation causes pulsus paradoxus
E - Hyperinflation compromises right ventricular function and left ventricular afterload is increased leading to drop in systolic blood pressure
E - Hyperinflation compromises right ventricular function and left ventricular afterload is increased leading to drop in systolic blood pressure
Hyperinflation in severe asthma has significant cardiac consequences. Pulsus paradoxus is exaggerated drop in blood pressure during inspiration. Normally, there is a drop of 10 mmHg in blood pressure during inspiration. Hyperinflation stretches the pulmonary vasculature, causing an increase in pulmonary vascular resistance and decrease right ventricular function. Hyperinflation also increases the left ventricular afterload especially during inspiration. This leads to exaggeration of drop of blood pressure during inspiration in severe asthma. Hence, option E is the correct. Inhaled ß2 agonists are known to cause diastolic hypotension; however, that does not explain the variation of blood pressures during different phases of the respiratory cycle. Thus, option A is incorrect. Option B is incorrect because, though children with asthma are usually dehydrated, pulsus paradoxus cannot be explained. In severe dehydration leading to shock, hypotension will be evident. Option C is incorrect as there is no evidence of pericardial inflammation in asthmatics. In pericardial effusion, there is limited space of the pericardial sac. During inspiration, there is an increase in the venous return and the right ventricle expands; however, bulges in the left ventricle, causing a drop in blood pressure, limits the venous return during inspiration, therefore causing greater than drop in blood pressure. Option D is incorrect as there is no evidence that asthma exacerbations lead to COPD. In adults, COPD exacerbation is an important cause of pulsus paradoxus.
A new urine marker has been identified for early detection of acute kidney injury (AKI). Your ICU is interested in participating in a multisite clinical study comparing patients with serum markers consistent with AKI and results on the new urine test. Assume that the serum markers are 100% accurate in identifying AKI. The following results are obtained:
- 80 patients with AKI confirmed by serum marker and with the urine marker
- 20 patients with AKI confirmed by serum marker and without the urine marker • 40 patients without AKI by serum marker test and with the urine marker
- 60 patients without AKI by serum marker test and without the urine marker
What is the negative predictive value of this test?
A. 40%
B. 60%
C. 67%
D. 75%
E. 80%
Answer: D - 75%
Negative Predictive Value = True Negative/(True Negative + False Negative) = 60/(60+20) = 0.75 = 75%
A 3-year-old is admitted to the PICU with ingestion of an unknown substance. She is awake and at her baseline level of mentation. Her vitals are as follows: HR 68; BP 65/24; RR 20; SPO2 99% on RA. On ECG, she has prolongation of the PR interval but a normal QT interval. She has capillary refill of about 3 seconds peripherally. A small dose of subcutaneous epinephrine results in improvement in her BP to 95/55 and increase in her HR to 110. Which one of the following substances is most like to be at fault?
A - Beta-blocker
B - Nondihydropyridine calcium-channel blocker
C - Dihydropyridine calcium-channel blocker
D - Tricyclic antidepressant
E - Clonidine
B - Nondihydropyridine calcium-channel blocker
The toxidrome described is that of a nondihydropyridine calcium-channel blocker (bradycardia, hypotension, sluggish capillary refill, normal mentation until hypotension results in altered mental status, PR prolongation with normal QT interval). If beta-blockers were at fault, epinephrine would not reverse symptoms. If dihydropyridine CCB were at fault, symptoms would more likely be hypotension with reflexive tachycardia. Tricyclic antidepressant would cause hypotension but likely prolonged QT interval as well as sedation and likely metabolic acidosis resulting in increased respiratory drive as a compensatory mechanism. Clonidine would result in similar symptoms to those seen but the patient would be expected to have sedation as an additional side effect.
A 10-year-old child is on high frequency oscillatory ventilator (HFOV) for 2 days because of pneumonia and hypoxemic respiratory failure. The ventilator settings are mean airway pressure (MAP) 34 cm of H20, delta P 60 Hertz 6, inspiratory time 33%. His FiO2 is 0.60. Arterial blood gas shows a pH of 7.2; PaO2 78 mmHg; Pa CO2 80 mmHg and base deficit of 1. CXR does not show a pneumothorax and there is 9-rib expansion. Which one of the following options will help bring the pH to 7.3?
A - Deflating the endotracheal tube cuff
B - Increasing the MAP to 36 cm of H20
C - Increasing FiO2 to 0.80
D - Increasing Hz to 8
E - Decreasing MAP to 32 cm of H20
A - Deflating the endotracheal tube cuff
Creating endotracheal tube cuff leak is an alternative way to enhance CO2 clearance. Deflating the endotracheal tube creates a path for the CO2 to escape outside of the endotracheal tube. Thus, option A is correct. Option B is incorrect, as increasing the MAP affects alveolar recruitment and oxygenation. At high levels, MAP may worsen hypercarbia because of hyperinflation. Option C is wrong, because increasing FiO2 will not have any effect on CO2 clearance. Option D is incorrect, because increasing Hz will actually worsen hypercarbia. Decreasing MAP will cause atelectasis and cause hypoxemia.
A 3-year-old girl who weighs 20 kg is admitted to PICU with diagnosis of status asthmaticus. She receives 5 mg/kg of aminophylline loading dose over 20 minutes. Blood level of aminophylline immediately after loading dose is 12 mcg/ml. Assuming no elimination or clearance has occurred, which one of the following values represents the correct volume of distribution of aminophylline in this patient?
A - 8.33 L
B - 0.42 L
C - 0.83 L
D - 60 L
E - Not able to determine from the given information
A - 8.33 L
The formula to calculate volume of distribution (Vd) is as follows: Vd = total amount of drug in the body (drug) / concentration of drug in the blood or plasma (Cp). In this patient total drug given is 5 mg x 20 (weight of the patient) is 100 mg. Substituting the values in the formula Vd = 100 / 12 = 8.33 L. Option A is the correct response because this is the value obtained from the formula. Option B is not correct. If it were to be volume of distribution/kg then Vd would have been 0.42 L/kg. Options C, D, and E are all incorrect.
Which one of the following factors will not alter a patient’s alveolar PO2?
A - Right-left cardiac shunt
B - High altitude
C - High carbohydrate diet
D - FiO2
E - Hypopnea
A - Right-left cardiac shunt
This question’s answer is described by the alveolar gas equation. The equation is as follows (where PAO2 = alveolar PO2, Atm = atmospheric pressure, PH2O = partial pressure of water vapor, PaCO2 = partial pressure of arterial CO2, R = respiratory quotient which is a constant determined by an organism’s CO2 eliminated per O2 consumed. R is increased toward 1 with high carbohydrate diet from a normal value of 0.8): PAO2 = FiO2 x (Atm - PH2O) - PaCO2/R. A right to left cardiac shunt will not affect alveolar PO2 but it will affect the A-a gradient.
In patients with heart failure, which of the following mediators that compose the neurohumoral response functions as a vasodilator?
A. Endothelin
B. Angiotensin II
C. Arginine vasopressin
D. Norepinephrine
E. B-type natriuretic peptide
Answer: E - B-type natriuretic peptide
In patients with heart failure, norepinephrine, angiotensin II, arginine vasopressin, and endothelin are vasoconstrictors that result in cardiac stimulation and fluid retention. B-type natriuretic peptide (BNP) is both a vasodilator and a natriuretic. The natriuretic peptides function as both hormones and neurotransmitters. There are 3 types: A-type, B-type, and C-type. BNP is secreted by the ventricular myocardium in response to elevated end-diastolic pressure and volume. It has diuretic, natriuretic, and venous and arterial vasodilation properties. The latter occur via cyclic guanosine monophosphate generation. BNP has been used as a diagnostic and prognostic marker in patients with congestive heart failure.
Which one of the following statements regarding respiratory quotient (RQ) is accurate?
a. It is the ratio at any given time of the volume of CO2 produced to the volume of O2 consumed
b. RQ of fat is 1.0 and carbohydrate is 0.7
c. RQ for protein is 0.6
d. RQ of brain is 0.97-0.99
e. RQ is affected by multiple factors including exercise/metabolic acidosis
d. RQ of brain is 0.97-0.99
The respiratory quotient is the ratio in the steady state of the volume of CO2 produced to the volume of oxygen consumed per unit of time. The RQ of carbohydrate is 1, and that of fat is about 0.7. Determining the RQ of proteins in the body is a complex process, but an average value of 0.82 has been calculated. RQ should be distinguished from the respiratory exchange ratio (R), which is the ratio of CO2 produced to O2 consumed at any given time whether or not equilibrium has been reached. R is affected by factors other than metabolism. The oxygen consumption and CO2 production of an organ can be calculated at equilibrium, by multiplying its blood flow per unit of time by the arteriovenous difference for O2 and CO2 across the organs, RQ of the brain is regularly 0.97–0.99, indicating that its principle but not its only fuel is carbohydrate.
In a pediatric patient who is intubated and on mechanical ventilation with SIMV volume control mode of ventilation, which one of the following options is the dependent control variable for the flow controller?
A - Pressure
B - Flow
C - Volume
D - Resistance of the ventilator circuit
E - Compliance of the lung
A - Pressure
During SIMV volume control, the mode is controlled by a flow controller. The dependent variable is pressure and the independent variable is flow. Volume is the limiting variable. Flow is determined by pressure divided by flow but does not affect the dependent variable. Compliance of the lung has no effect on the control variable but may affect the limit variable.
With regard to hemoglobin oxygen dissociation curve, which one of the following statements is correct?
A - The curve shifts to the left with an increase in temperature
B - Oxygen affinity of hemoglobin increases with acidosis
C - The curve shifts to the right with an increase in thyroid hormones
D - There is a decrease in p50 with exercise
E - The curve shifts to the left with increase in HbF
C - The curve shifts to the right with an increase in thyroid hormones
The oxygen dissociation curve relates to the percent saturation of oxygen carrying power of Hb. It has a characteristic sigmoid shape. Depending on the affinity of Hb for oxygen, the curve shifts to the right or to the left. Affinity of Hb for oxygen is affected by multiple physiologic variables. A shift to right is associated with decrease in affinity. A convenient index of such shifts is p50, the pO2 at which Hb is half-saturated with oxygen. The higher the p50, the lower the affinity. Oxygen dissociation curve is shifted to the right (decrease in affinity, increase in p50) with fever, acidosis, and an increase in BPG. Other factors shifting the curve to the right include the following: thyroid/growth hormone/androgens (by increasing concentration of BPG); exercise (increase in p50 as temperature rises in active tissues and CO2 accumulates, lowering pH); high altitude (increase in BPG); decrease in HbF (HbF has high affinity, which facilitates the movement of oxygen from mother to fetus); anemia (increase in BPG); abnormal Hb (high oxygen affinity causes tissue hypoxia, increased PRBC production, polycythemia). BPG decreases in stored blood, so PRBC transfusion lead to increased oxygen affinity and less tissue oxygen delivery.
Which one of the following pathogens is a common cause of necrotizing fasciitis in children?
a. Group-A Streptococcus (GAS)
b. Streptococcus pyogenes
c. Staphylococcus aureus
d. E. coli
e. Pseudomonas
a. Group-A Streptococcus (GAS)
Necrotizing fasciitis is a deep seated infection of the subcutaneous tissue that rapidly invades and destroys underlying fascia and fat despite often sparing the overlying skin and muscle. It is commonly caused by GAS; other inciting pathogen includes clostridium perfringens and clostridium septicum.
Which one of the following statements is not accurate regarding case-control studies?
A - Decreasing the number of cases strengthens the study
B - Cases are identified before the controls
C - Helpful to study rare diseases
D - Increasing the number of cases strengthens the study
E - Case and controls are identified simultaneously
A - Decreasing the number of cases strengthens the study
Case-control studies are ideal for rare diseases, as conducting a randomized controlled trial in such a disease would not be feasible (making option C incorrect). In case-control studies, cases are selected before the controls—hence, options B and E are incorrect. Increasing the number of cases increases the power of the study; however, this may the limiting step in case-control studies. Hence, option D is incorrect while option A is correct.
A 6-month-old boy with a single ventricle that was previously palliated with an aortopulmonary shunt returns from surgery after undergoing a bidirectional cavopulmonary anastomosis. His oxygen saturation is 65%. What is the most likely cause for the systemic desaturation?
A. Pulmonary arteriovenous malformations
B. Decompressing venous collaterals
C. Atrioventricular valve regurgitation
D. Increased pulmonary vascular resistance
Answer: D - Increased PVR
Possible causes of hypoxemia following a bidirectional cavopulmonary anastomosis can be grouped into 3 categories: pulmonary venous desaturation, systemic venous desaturation, and decreased pulmonary blood flow. Decompressing venous collaterals are one cause of decreased pulmonary blood flow, along with undiagnosed contralateral superior vena cava (SVC). In bidirectional cavopulmonary circulation, any increase in pulmonary vascular resistance can open decompressing veins, resulting in a shunt of venous blood into the heart. Factors related to development of decompressing venous collaterals include bilateral SVC, elevated SVC pressure, and high early postoperative transpulmonary gradient. Vessels may be closed on the pre-Glenn cardiac catheterization, but a left SVC-to-coronary sinus collateral may develop postoperatively.
Pulmonary arteriovenous malformations typically cause hypoxemia months to years after the surgery. Atrioventricular valve regurgitation is less likely to affect systemic oxygenation. Increased pulmonary vascular resistance causes systemic desaturation.
A newborn underwent full correction of interrupted aortic arch (type 1B) and right aortic arch at 1 week of age and recovered uneventfully. Postoperative evaluation by echocardiography showed normal function, trivial residual ventricular septal defect, and no arch gradient. The infant is now 3 weeks old and has had difficulty with feeding due to poor coordination of suck and swallow. The infant is receiving nasogastric tube feeds. Current medications include furosemide, 2 mg/kg every 8 hours, and aspirin, 41 mg daily, both via nasogastric tube. You are called urgently to evaluate him because the ward team observed a 2-minute generalized seizure. He received a dose of midazolam, 0.1 mg/kg IV, and bedside glucose measurement indicated no hypoglycemia (90 mg/dL). The infant is in no distress as HR is 140/min, RR is 30/min, BP is 65/35 mm Hg, and SaO2 is 100%. The most appropriate immediate step in investigation for an etiology for the seizure includes:
A. Arranging electroencephalography
B. Doing an emergency head ultrasonography
C. Ordering chest radiography to rule out aspiration
D. Sending a formal blood glucose sample to the lab
E. Sending blood for serum electrolyte and ionized calcium values
Answer: E - Send blood for serum electrolytes and ionized calcium
This patient has DiGeorge syndrome, which is commonly associated with hypoparathyroidism and hypocalcemia. The seizure may or may not be related to hypoglycemia; however, in this scenario, low blood sugar has been ruled out and a correctable cause of seizures would be to investigate the possibility of hypocalcemia. Head ultrasonography and electroencephalography, although important to delineate the underlying etiology, will not direct acute interventions. Chest radiography to rule out aspiration is not indicated since the patient has no respiratory distress.
Which of the following conditions is represented on the monitor?
A. Hypovolemia
B. Ebstein anomaly
C. Junctional ectopic tachycardia
D. Tricuspid regurgitation
Answer: C - Junctional ectopic tachycardia
An atrial tracing has 3 positive waves, a, c, and v, and 2 negative descents, x and y, as shown in Figure B below. The a wave is caused by atrial contraction or systole, the c wave is caused by ventricular contraction or tricuspid valve closure, and the v wave by atrial filling against a closed tricuspid valve. Atrial pressure is commonly monitored following cardiac surgery; analysis of the waveforms can provide important information about cardiac function.
Large or cannon a waves occur when the atrium contracts against a closed or obstructed atrioventricular valve or when there is resistance to ventricular filling. Structural lesions that can produce cannon a waves include tricuspid valve stenosis, pulmonary stenosis, pulmonary atresia, pulmonary hypertension, or a noncompliant right ventricle (due to hypertrophy or diastolic dysfunction). Arrhythmias associated with atrioventricular dyssynchrony can also produce cannon a waves; these include complete heart block and junctional ectopic tachycardia.
Enlarged v waves result when ventricular pressure is transmitted to the atria as seen with tricuspid regurgitation, Ebstein anomaly, or a left ventricle–to–right atrial shunt. Correlation of the observed waveforms on the ECG tracing can help distinguish a waves from v waves.
A 6-month-old boy is admitted to the pediatric ICU 1 hour after presenting to the clinic with a 2- week history of poor feeding and pallor. A session of acupuncture was done the day before admission and the parents both believe that it helped him to feel better. He has episodic tachypnea (RR of 30-65/min), a temperature of 37.8°C (100°F), HR of 175/min, and BP of 75/46 mm Hg. The lung fields are clear to auscultation, with normal heart sounds. The abdomen is full with adequate bowel sounds and mild erythema at the umbilicus. The extremities are warm and the capillary refill is less than 2 seconds. Past medical history is significant for chronic omphalitis and a bone marrow examination showing a paucity of neutrophils. Laboratory findings reveal a hemoglobin level of 6.4 g/dL, hematocrit of 19%, WBC count of 13,500/μL (0% neutrophils, 45% lymphocytes), sodium level of 134 mEq/L, potassium level of 4.6 mEq/L, and total carbon dioxide level of 23 mEq/L. Which of the following next steps is most appropriate?
A. Administer recombinant human granulocyte colony-stimulating factor (G-CSF), which may improve the patients’ quality of life
B. Consider a 10-mL/kg bolus of fluid with packed red blood cell transfusion and simultaneous infusion of IV antibiotics that includes ceftriaxone
C. Initiate resuscitation with high-flow nasal cannula oxygen followed by a fluid bolus of 30 mL/kg within 15 minutes according to the Surviving Sepsis guidelines.
D. Obtain an interleukin-8 assay to evaluate the immune response of the patient relative to the capacity of the infiltrative cells
E. Order a bone marrow aspiration to confirm a diagnosis of Kostmann syndrome (severe congenital agranulocytosis).
Answer: B
This case highlights the dilemmas that clinicians may encounter when faced with a patient with immunosuppression and severe anemia in the era of implementation of sepsis protocols. In addition, the knowledge about severe congenital agranulocytosis as a cause of immunodeficiency and weighing the value of therapeutic interventions is highlighted.
Neutrophils are a major contributor to the infiltrative capacity of the innate immune response and also produce interleukin-8. However, an assay is not essential in the initial management of this patient who is presenting with signs of sepsis.
The implementation of high-flow nasal cannula oxygen is highly recommended for the initial resuscitation of a patient who presents with severe sepsis. In addition, fluid resuscitation is recommended up to 20 mL/kg over 5-10 min, targeting improved perfusion, resolution of urine output, and amelioration of mental status. In this case, there are other factors contributing to the hemodynamic changes that should be considered: the severe anemia and the elevated temperature. Since the capillary refill is less than 2 seconds, aggressive fluid resuscitation could tip the balance and drown the patient into fluid overload and pulmonary congestion in the face of possible high-output failure from the chronic anemia.
The early administration of granulocyte colony-stimulating factor (G-CSF) has been known to improve the immunologic condition of these patients and help them to lead a normal life. However, this is not a critical step in the acute management of the patient at this time.
The absence of neutrophils in the peripheral smear and a bone marrow sample devoid of neutrophil precursors is diagnostic for Kostmann syndrome
Resuscitation with a blood transfusion in cases of severe anemia is considered to be superior to crystalloid or albumin and should be considered as a critical initial step in the management of this patient who is presenting with systemic inflammatory response syndrome and sepsis that is likely to progress to septic shock.
A new urine marker has been identified for early detection of acute kidney injury (AKI). Your ICU is interested in participating in a multisite clinical study comparing patients with serum markers consistent with AKI and results on the new urine test. Assume that the serum markers are 100% accurate in identifying AKI. The following results are obtained:
- 80 patients with AKI confirmed by serum marker and with the urine marker
- 20 patients with AKI confirmed by serum marker and without the urine marker • 40 patients without AKI by serum marker test and with the urine marker
- 60 patients without AKI by serum marker test and without the urine marker
What is the sensitivity of the test in this study?
A. 40%
B. 60%
C. 67%
D. 75%
E. 80%
Answer - E - 80%
Sensitivity is the fraction of people with acute kidney injury(AKI) (serum marker positive) who test positive with the urine test. Specificity is the fraction of people without AKI (serum marker negative) who test negative with the urine test. Positive predictive value is the fraction of the people who test positive with the urine test who actually have AKI (serum marker positive). Negative predictive value is the fraction of people who test negative for the urine test who do not have AKI (serum marker negative).
Which one of the following choices indicates the need for permanent pacing in acquired atrioventricular (AV) block in a pediatric population?
A - Advanced second- or third-degree AV block with low cardiac output, congestive heart failure (CHF), or symptomatic bradycardia
B - Asymptomatic sinus node dysfunction during age-inappropriate bradycardia
C - Congenital third-degree AV block without ventricular dysfunction and with a normal QRS
D - Congenital third-degree AV block in child ≥1 year of age with an average heart rate of 65 bpm
E - Long-term treatment with digitalis for bradycardia-tachycardia syndrome
A - Advanced second- or third-degree AV block with low cardiac output, congestive heart failure (CHF), or symptomatic bradycardia
This question describes the most accurate indication for permanent pacing in AV block in a pediatric population. Option A is correct because according to the American Heart Association (AHA) guidelines, pediatric patients with second- or third-degree AV block with low cardiac output, CHF, or symptomatic bradycardia are ideal candidates for permanent pacing. Option B is incorrect because sinus node dysfunction during age-inappropriate bradycardia that is correlated with symptoms is an indication for permanent pacing. Option C is incorrect because congenital third-degree AV block is an indication for permanent pacing when there is ventricular dysfunction or a wide QRS escape rhythm. Option D is incorrect because permanent pacing is indicated in congenital third-degree AV block in children 1 year of age or older who have an average heart rate less than 50 bpm. Option E is incorrect because long-term treatment with digitalis for bradycardia-tachycardia syndrome is not an indication for permanent pacing; if long-term treatment is required with another agent, then permanent pacing may be indicated.
Which one of the following tests is most appropriate to confirm the initial diagnosis of Lyme disease?
A - Serological testing for antibodies
B - Biopsy of the rash
C - Appearance of erythema migrans is sufficient
D - Lumbar puncture
E - Blood studies to determine WBC
E - Blood studies to determine WBC
C - Appearance of erythema migrans is sufficient
This question describes the most appropriate diagnostic test to confirm infection with B. burgdorferi. Because the child is in the early stage of Lyme disease and erythema migrans is present, option A is incorrect. Serological testing for antibodies is often negative during early stages of Lyme disease and would not provide a differential diagnosis. Option B is incorrect as biopsy of the rash would not provide a differential diagnosis. Option C is correct because the presence of erythema migrans is highly specific to infection with B. burgdorferi and Lyme disease. Option D is incorrect because lumbar puncture is only indicated in some patients with late stage Lyme disease with cranioneuropathy. Option E is incorrect because the WBC may or may not be elevated in Lyme disease patients and therefore adds no diagnostic value.
A 4-year-old, 22-kg boy with influenza pneumonia develops worsening respiratory failure and requires intubation and mechanical ventilation for severe hypoxemia. It’s noted that he is being ventilated in volume-control SIMV mode with a rate of 20 (which he is overbreathing to 22), a tidal volume of 220 ml (with resultant plateau pressures of 32-35), an end expiratory pressure of 10 cm H2O, and an FiO2 of 1. His recent ABG indicates a pH of 7.41, PaCO2 of 32, and PaO2 of 58. Which one of the following changes in management would most likely reduce his mortality on the basis of existing animal and adult clinical data on ARDS?
A - Transition to high-frequency oscillating ventilation
B - Increase in positive end-expiratory pressure to 16 to 20 cm H2O to optimize recruitment
C - Sedation and paralysis (if needed) to reduce his overbreathing and create mild hypercapnia
D - Extracorporeal membrane oxygenation
E - Reduction of tidal volumes to reduce plateau pressures
E - Reduction of tidal volumes to reduce plateau pressures
In a large randomized trial of adult ARDS patients, low tidal volume ventilation (6 ml/kg) has been proven to reduce mortality compared to higher tidal volumes (12 ml/kg). Additional animal and clinical studies have correlated ventilator induced lung injury with excessive tidal volumes and plateau pressures. No one mode of ventilation is known to be superior for management to another (e.g., HFOV is not better than conventional). Although a recent adult study did reveal a mortality benefit from 24-hour neuromuscular blockade, this was in conjunction with low tidal volume ventilation and not due to the effect of respiratory rate or CO2. Transfer to an ECMO center in the UK provided superior outcomes in a recent adult ARDS trial in the UK, but this result was present in both patients who received ECMO and those who did not; thus, bringing into question whether it was in fact additional management strategies, such as low tidal volume ventilation, which underlay the benefit.
Concentration of which one of the following substances is higher in CSF, as compared to plasma?
a. Protein
b. Glucose
c. Potassium
d. Chloride
e. Cholesterol
D - Chloride
The following table compares the concentration of various substances in human CSF and plasma:
Identify endothelial cells, fixed macrophages, capillaries, and interalveolar pores (of Kohn) in a histological slide of an alveolus
Endothelial cells (E) – pulmonary capillaries
Fixed macrophages (M)
Alveolar macrophages – motile, in alveolar lumen
RBC and WBC’s in the capillaries (C)
Interalveolar pores (of Kohn)
- Through the common alveolar septum;
- Equilibrates the air pressure in adjacent alveolar spaces
A 2-year-old child has a history of recurrent attacks of acute onset edema of tongue/throat with stridor, without urticaria. This edema happens without any precipitating factor; swelling gradually worsens over 12-36 hours and then subsides over a period of days. He has a family history of similar episodes in his father and paternal uncle. Which one of the following options is a good screening test to diagnose his condition?
A - Serum C3 level
B - Serum C4 level
C - Serum C1 level
D - Absolute eosinophil count
E - Histamine challenge test
B - Serum C4 level
Hereditary angioedema (HAE) is a rare, autosomal dominant disorder of C1 inhibitor (CIINH) deficiency. CIINH deficiency presents in adolescent or early childhood with recurrent attacks of submucosal or subcutaneous edema. The edema and swelling gradually worsens for 12-36 hours and then subsides over a period of days. Edema of the upper respiratory tract, which can variably affect some or all of the tongue, pharynx and larynx, can be life threatening. Submucosal edema of the G.I tract can present as abdominal pain, nausea and vomiting. Urticaria is not a presenting feature of CIINH deficiency. Serum C4 level has been recommended as a screening test for CIINH deficiency, as serum C4 is invariably low in untreated HAE (C4
Which one of the following muscles is the most important for forced expiration?
a. Diaphragm
b. Sternocleidomastoid
c. External Intercostal
d. Internal Intercostal
e. Pectoralis major
d. Internal Intercostal
Respiratory muscles and movement of the diaphragm accounts for 75% of the change in intrathoracic volume during quiet inspiration. The other important inspiratory muscles are the external intercoastal muscles. The scalene and sternocleidomastoid muscles in the neck are accessory inspiratory muscles that help to elevate the thoracic cage during deep, labored respiration. The main expiratory muscles are the internal intercoastal. The internal intercostals have this action because they pass obliquely downwards and posteriorly from rib to rib and therefore pull the ribcage downwards when they contract. Contractions of the muscles of the anterior abdominal wall also aid expiration by pulling the ribcage downwards and inwards and by increasing the intraabdominal pressure, which pushes the diaphragm upward.
Which one of the following factors is reduced in neonates which affects drug absorption?
A - Intestinal motility
B - Proportion of water in body weight
C - Plasma half-lives for many drugs
D - Circulating free fatty acids
E - Circulating bilirubin
A - Intestinal motility
This question describes one factor that is reduced in neonates which affects drug absorption. Option A is correct because intestinal motility is one factor that is reduced in neonates which affects drug absorption and should be taken into consideration when determining proper dosage for a drug. Option B is incorrect because neonates have the greatest proportion of water to body weight as compared to other ages. Option C is incorrect because plasma half-lives for many drugs are increased in neonates. Options D and E are incorrect because there are greater levels of circulating free fatty acids and bilirubin in neonates as compared to adults.
Which of the following would result in an acute decrease in SVO2 in cardiogenic shock?
A. Decreased stroke volume
B. Inotropic therapy
C. Transfusion of PRBCs
D. Vasodilator therapy
Answer: A - Decreased stroke volume
SVO2 serves as a surrogate marker for cardiac output. Factors that decrease cardiac output will result in increased arteriovenous DO2 difference and decreased SVO2. These would be factors such as decreased stroke volume, anemia, and increased afterload. Vasodilator therapy and therefore decreased afterload would lead to improved cardiac output and a corresponding increase in SVO2.
SvO2 is assumed to reflect the balance between arterial oxygen delivery (DO2) and oxygen consumption (VO2) provided arterial blood oxygen saturation (SaO2) is normal. The modified
Fick equation states:
SvO2 = SaO2 – [VO2/(Cardiac Output × Hemoglobin × 1.34)]
Care must be taken when interpreting SvO2 in shock states. In any shock state, oxygen demand exceeds VO2 by definition such that SvO2 cannot reflect the balance between DO2 and oxygen demand. Second, as VO2/DO2 dependency is a characteristic pattern of shock states, any increase in DO2 during resuscitation will be associated with a simultaneous increase in VO2 and hence with no or only a small increase in SvO2 until a critical DO2 is reached. SvO2 should not change much in response to an increase in cardiac output in cases of hyperdynamic shock states.
Interest in SvO2 monitoring, SvO2 surrogate, and central venous oxygen saturation, in septic shock is recommended. SvO2 has been recommended as a major hemodynamic target of early resuscitation in septic shock.
A 6-year-old child with renal failure and DIC is on continuous venovenous hemofiltration (CVVH). Due to risk of bleeding with heparin, a commercial citrate infusion is being used. Which one of the following is an SE of citrate anticoagulation?
A - Hypercalcemia
B - Metabolic acidosis
C - Metabolic alkalosis
D - Hypokalemia
E - Hyponatremia
C - Metabolic alkalosis
Citrate anticoagulation is based on the concept of chelation of the calcium in order to avoid clotting of blood in the chronic renal replacement therapy (CRRT) circuit. Complications of citrate anticoagulation include hypocalcemia, metabolic alkalosis, and citrate toxicity. Metabolic alkalosis develops due to hepatic conversion of 1 mol of citrate to 3 mol of bicarbonate.
A 5-year-old boy presents to the emergency department with altered mental status after a motor vehicle collision. He is on a backboard and a cervical collar is in place. HR is 62/min, and he is being bag-ventilated at a rate of 15/min with an oxygen saturation of 98% on 100% oxygen. He has good chest rise with bagging. His BP is 152/89 mm Hg and he has unequal pupils. The most appropriate next step in his management is to:
A. Have ketamine, atropine, and rocuronium drawn up for intubation
B. Initiate intentional hyperventilation at 25/min with bag-valve-mask while preparing for intubation
C. Perform a tertiary survey
D. Place a roll under the shoulders and hyperextend the neck for better airway alignment and visualization
Answer: B
The child is showing signs of increased intracranial pressure and should have immediate hyperventilation while preparing for intubation. Ketamine is relatively contraindicated in patients with elevated intracranial pressure. Additionally, while one may remove the cervical collar to facilitate intubation, cervical spine immobilization should be maintained at all times. Tertiary survey is not performed until a patient has been stabilized.
A 7-year-old boy with pneumonia is intubated and is on mechanical ventilator. His current settings are: mode: pressure control, PIP above PEEP 15, PEEP5, FiO2 50%, rate 25. On these settings, his exhaled tidal volume is 200 ml (6.5 ml/kg). His arterial blood gas on these settings are pH 7.13/PaCO2 70, PaO2 80. Patient is sedated and paralyzed. In order to decrease the PaCO2 to 50 mmHg, PIP needs to be increased to which one of the following levels?
A - 18
B - 21
C - 24
D - 27
E - 30
B - 21
Desired Vt = known PaCO2 × known Vt / desired PaCO2. Desired Vt = 70 × 200 / 50 = 280 ml. A pressure change (ΔP) of 15 results in a Vt of 200 ml, A ΔP than will generate tidal volume of 280 ml, can be estimated by first calculating static compliance. Static compliance Cs = Vt / ΔP. Cs = 200/15 = 13 ml/cm H2O. If compliance remains the same: Desired ΔP = desired Vt / Cs. Desired ΔP = 280/13. Desired ΔP = 21.
Which one of the following options is a characteristic of intraparenchymal catheters as compared to intraventricular systems?
A - Less invasive
B - More accurate
C - The property of “zero drift”
D - Less expensive
E - Higher risk of infections
C - The property of “zero drift”
There are 2 types of ICP monitors currently in use: intraparenchymal catheters and the intraventricular systems. A ventricular catheter connected to an external monitoring devise, is the most accurate and low-cost method for ICP monitoring. This method has proven to be reliable and permits periodic re-zeroing; it also allows the benefit of therapeutic CSF drainage. However, currently the most common location for ICP monitoring is the cerebral parenchyma. Contemporary intraparenchymal transducers may be classified as solid-state based on silicon chip with pressure sensitive resistors, or of fiberoptic design. Although both systems are very accurate at the time of placement, they have been reported to zero-drift over time, which can result in an error after 4-5 days.
A 2-week-old male child, with history of tachypnea and poor feeding, was admitted to PICU with a diagnosis of bronchiolitis. Patient is on mechanical ventilation for respiratory distress with positive end- expiratory pressure (PEEP) of 8; FiO2 of 0.7; and pulse oximeter oxygen saturation is 88%-95%. On examination, his heart rate is 160/minute, blood pressure is 65/35 mm kg, and capillary refill is 2-3 seconds. He has fixed split second sound; no cardiac murmur. His liver is enlarged 4 cm below the costal margin. His urine output is 2 ml/kg/hr. His arterial blood gas shows pH of 7.34, PaCO2 of 40, PaO2 of 56, base excess of negative 2. Rapid test for respiratory syncytial virus is negative. His chest X-ray is shown below. Which one of the following courses of action is the most appropriate for this child?
a. Fluid bolus followed by prostaglandin (PGE1) infusion to open ductus arteriosus
b. Start iNO at 20 parts per million and increase PEEP to 10
c. Echocardiography followed by surgery as soon as possible
d. Start continuous milrinone infusion, followed by epinephrine infusion
e. Place child on venoarterial extracorporeal membrane oxygenation (ECMO)
c. Echocardiography followed by surgery as soon as possible
This is a child with supracardiac total anomalous venous return (TAPVR). His chest X-ray has classic snowman appearance. This patient has pulmonary venous congestion with right heart failure. His hemodynamic status seems acceptable with no significant acidosis. Option A is incorrect because prostaglandin infusion will increase pulmonary blood flow and should be avoided. Option B is also incorrect because iNO can increase pulmonary blood flow and will worsen the situation. This patient may need iNO after surgery but not indicated during preoperative period. Option C is correct response because echocardiography is usually enough to diagnose total anomalous venous return TAPVR in this patient. TAPVR patients require surgical correction as soon as possible. Option D is incorrect because milrinone can increase pulmonary blood flow and epinephrine is not needed due to acceptable hemodynamic status. Option E is incorrect because this is a relatively stable patient and there is no need to place this patient on ECMO before surgery. Some of the TAPVR patients may require ECMO after surgery, more so if they have associated hypoplastic left ventricle.
A 6-year-old girl who weighs 15 kg (33 lbs) and is 100 cm (39 in) in length is admitted to the pediatric ICU with pneumococcal sepsis. Her serum creatinine level on admission is 0.4 mg/dL, and it rises to 1.5 mg/dL 2 days later. Which of the following best characterizes estimated glomerular filtration rate at this time?
A. >50 mL/min/1.73 m2; unchanged from normal baseline
B. Between 10 and 50 mL/min/1.73 m2; 75% of baseline
C. Between 5 and 10 mL/min/1.73 m2; 50% of baseline
D.
Answer:D -
Serum creatinine and creatinine-based equations can only be used to assess glomerular filtration rate (GFR) when a steady state exists. In the setting of acute kidney injury, an abrupt decline in GFR is associated with a gradual rise in serum creatinine until a new steady state is achieved. Use of the serum creatinine level to estimate the GFR in this period will result in an overestimation of the renal function. A doubling of serum creatinine corresponds to an approximate 50% decrease in remaining renal function. Therefore, her nearly 4-fold increase in serum creatinine suggests that her GFR is less than 30% of normal. Serum creatinine levels should be monitored once or twice a day to better estimate her GFR.
What is cardiac output?
HR x SV
Osmotherapy is most effective in which one of the following forms of cerebral edema?
a. Cytotoxic
b. Vasogenic
c. Interstitial
d. Equally not effective in all forms
e. Equally effective in all forms
a. Cytotoxic
Cerebral edema is conventionally classified into vasogenic, cytotoxic, and interstitial edema based on its underlying pathogenesis. Vasogenic cerebral edema occurs due to disruption of blood brain barrier. Increased capillary permeability due to blood brain barriers disruption allows intravascular fluid and solutes to enter brain interstitial fluid, causing its expansion. Cytotoxic cerebral edema occurs due to accumulation of higher amount of intracellular salute compared to extracellular fluid, which leads to movement of water into the cells to maintain osmotic equilibrium. Diseases which impair cellular metabolism (hypoglycemia, rye syndrome) or sustained hyperosmolar state (hypernatremia, hyperglycemia) are associated with cytotoxic edema. Interstitial cerebral edema occurs when there is increased hydrostatic pressure gradient between the ventricle system and the brain interstitium, resulting in transependymal movement of CSF. Interstitial endemia is seen in obstructive and nonobstructive hydrocephalus. Consideration of pathophysiologic mechanism in cerebral edema has important therapeutic significance. Osmotic therapy is most effective in cytotoxic cerebral edema, whereas relief of CSS obstruction is necessary to treat interstitial cerebral edema. It is, however, important to realize that all 3 forms of cerebral edema can occur concurrently in the same patient.
Which of the following letters in the figure refers to the closing capacity?
A. A
B. B
C. C
D. D
E. E
Answer: C
The closing capacity is defined as the closing volume plus the residual volume. In the figure, the residual volume is represented by the letter B. The closing volume is not depicted in the figure, but represents some portion of the expiratory reserve volume. The closing capacity exceeds functional residual capacity (FRC) (letter E) when some lung segments are closed during tidal breathing. This leads to hypoxia and hypercapnia. If the closing capacity exceeds both the FRC and tidal volume (letter A), lung segments will remain closed during both inspiration and expiration. This is important clinically and represents areas of atelectasis. Opportunities to raise FRC above closing capacity by administering positive pressure in the form of positive end- expiratory pressure or continuous positive airway pressure are useful. Children younger than 6 years have a closing capacity greater than FRC when supine.
A 5-year-old who presents in status asthmaticus required intubation approximately 30 minutes earlier due to obtundation and severe hypercarbia (PaCO2 = 78). Rocuronium and ketamine were used at the time of intubation, and the child is now on fentanyl 1 mcg/kg/hour infusion for sedation. When called to evaluate him for hypotension, it’s noted that he’s quite agitated, with HR of 142, RR of 54 (vent set rate 18), and BP of 62/45. His oxygen saturations are 98% with FiO2 of 0.4, and he has diffuse wheezing bilaterally. Which one of the following next steps will best address the underlying cause of his hypotension?
A - Order a stat chest radiograph
B - Needle decompression of the chest
C - Disconnect the patient from the mechanical ventilator followed by sedation and/or paralysis
D - Administer a fluid bolus and begin an epinephrine infusion
E - Administer nebulized albuterol
C - Disconnect the patient from the mechanical ventilator followed by sedation and/or paralysis
This is a classic situation where auto-PEEP, due to tachypnea in the setting of airflow obstruction (bronchospasm), results in shock by impairing venous return. Given the severity of the shock, there is a need for rapid action to permit exhalation, which in this case would come from disconnection from the vent circuit followed by sedation. In addition, paralysis (if needed) would reduce the respiratory rate and permit adequate exhalation. While a tension pneumothorax may result in hypotension, it is unlikely to be present in this scenario due to the presence of bilateral breath sounds. A fluid bolus could help the hypotension but does not address the underlying etiology. Beta agonist therapy is necessary to combat the bronchospasm, but will not act sufficiently quickly to reverse the patient’s shock in this case.
Regarding emergency airway management in children, which one of the following statements is the most accurate?
a. Succinylcholine can be safely used in children with spinal cord injuries
b. Laryngeal mask airway can be safely used in patient with full stomach
c. Blind nasotracheal intubation is recommended for conscious children with multiple traumas
d. Orotracheal intubation with sedation, analgesia, and nondepolarizing paralytics is preferred method
e. Primary tracheotomy without any attempts of intubation is safe in children with cervical spine injuries
d. Orotracheal intubation with sedation, analgesia, and nondepolarizing paralytics is preferred method
Safe management of airway is of paramount importance in acute emergencies. Option A is incorrect. Patients with spinal cord injuries are at risk for extreme hyperkalemia following administration of succinylcholine. Option B is incorrect. Laryngeal mask airway is not the ideal technique to use in a patient with full stomach because its design does not prevent aspiration of gastric contents. Option C is incorrect because current advanced trauma support guidelines no longer recommend blind nasotracheal intubation. Option D is the correct response. Orotracheal intubation with sedation, analgesia, and nondepolarizing neuromuscular blocking agents is safer in children. In suspected cervical spine injury, immobilization of head and neck in neutral position by an assistant is necessary. Option E is incorrect. Data does not support either the necessity or safety of routinely using primary tracheotomy before attempting intubation.
A 3-year-old child with a single left ventricle underwent a fenestrated Fontan procedure. He returns from the cardiac catheterization lab with pulmonary artery pressure of 20 mm Hg, left atrial pressure of 5 mm Hg, and systemic oxygen saturation of 99%. There is no pressure gradient between a simultaneous pulmonary artery wedge pressure and an end-diastolic pressure of the ventricle of 4 mm Hg. Which of the following assessments of the patient’s physiology is most accurate?
A. The fenestration is patent
B. The transpulmonary gradient is normal
C. The pulmonary vascular resistance is normal
D. There is no evidence of atrioventricular valve or pulmonary venous stenosis
Answer: D - There is no evidence of AVV or pulmonary venous stenosis
The catheterization data show an elevated transpulmonary gradient (pulmonary artery pressure minus left atrial pressure) of 15 mm Hg. Therefore, the patient is likely to have elevated pulmonary vascular resistance. The fenestration probably is not patent because the oxygen saturation is 99%, despite a pulmonary artery pressure of 20 mm Hg. With a pulmonary artery pressure of 20 mm Hg, some shunting of venous blood should be expected through a patent fenestration. Hence, the systemic oxygen saturation would be less than 99%. Simultaneous pulmonary artery wedge and end-diastolic pressure shows no gradient, evidence that there is no stenosis of the atrioventricular valve or pulmonary venous anatomy.
According to pediatric guidelines for severe traumatic brain injury (TBI), the target cerebral perfusion pressure (CPP) for a 2-year-old, 15-kg (33-lb) child should be maintained above which of the following minimum levels?
A. 30 mm Hg
B. 40 mm Hg
C. 50 mm Hg
D. 60 mm Hg
Answer: B - 40 mmHg
Cerebral perfusion pressure (CPP) is the difference between mean arterial pressure and mean intracranial pressure (ICP). In children, a CPP of less than 40 mm Hg is associated with increased mortality. Although the optimal values are unknown, the pediatric guideline for treating severe traumatic brain injury is maintaining a CPP of more than 40 mm Hg in an adolescent. A modification to this recommendation is to titrate the CPP threshold according to age, with age-specific thresholds for children less than a year of age and adolescents.
A 3-year-old child involved in a motor vehicle collision (MVC) sustains an injury at T7. He is initially in no respiratory distress. But on hospital day 2 develops respiratory distress. Which one of the following muscle groups is important for active expiration in this child?
A - Deep intercostals
B - Outer intercostals
C - Diaphragm
D - Trapezius
E - Sternocleidomastoid
A - Deep intercostals
Internal or deep intercostals are the primary muscle of expiration. They are located between ribs deep and oriented at a right angle to external intercostals, continuous with internal obliques. The internal intercostals function to depress the ribs during expiration. The accessory muscles of expiration are the rectus abdominis, external obliques, internal obliques, transversus thoracis, serratus posterior inferior, quadratus lumborum. The diaphragm and outer or external intercostals are the major muscles in inspiration. The trapezius and sternocleidomastoid muscles are accessory muscles of inspiration.
A 1-day-old, full-term neonate has cyanosis, tachypnea, and an oxygen saturation of 82% in room air by pulse oximetry. Chest radiography shows streaky bilateral interstitial opacities. Cyanotic congenital heart disease is suspected. A hyperoxia test is performed with PaO2 levels obtained on 100% oxygen. Which of the following results is suggestive of cyanotic congenital cardiac disease?
A. PaO2 30 mm Hg before, 250 mm Hg after
B. PaO2 30 mm Hg before, 80 mm Hg after
C. PaO2 150 mm Hg before, 150 mm Hg after
D. PaO2 150 mm Hg before, 250 mm Hg after
Answer: B - PaO2 30mmHg before and 80mmHg after
An arterial PaO2 less than 100 mm Hg on 100% oxygen in the absence of obvious lung disease (“failed” hyperoxia test) is most likely due to intracardiac right-to-left shunting and is virtually diagnostic of cyanotic congenital heart disease. The increased PaO2 in the remaining answers is most likely related to variable responses to parenchymal lung disease.
A 3-month-old boy is being evaluated for the onset of hypotonia and feeding difficulties. His mother also reports he has a history of constipation. On examination, he is afebrile with normal growth parameters. Neurologic examination is significant for decreased muscle tone and a decreased gag reflex. His pupils are 4 mm bilaterally and sluggish to light. Deep tendon reflexes are 2+ bilaterally in upper and lower extremities. The remainder of his physical examination is unremarkable. A lumbar puncture reveals normal cell counts and chemistry results. Which of the following tests would be most helpful in establishing the diagnosis?
A. Brain MRI
B. Electroencephalography (EEG)
C. Stool toxin assay
D. Sweat chloride level
E. Thyroid-stimulating hormone test
Answer: C - Stool toxin assay
This patient has infantile botulism. Botulinum toxin is one of the most potent poisons known. Clostridium botulinum is a spore-forming, anaerobic, gram-positive bacillus with 8 known toxins. Botulinum toxin acts to prevent acetylcholine release, which decreases cholinergic activity in the peripheral nervous system. The central nervous system remains unaffected. Infantile botulism was first described in 1976. Typically, this disease affects infants 1-3 months of age. A common presentation involves constipation, feeding problems, and floppiness. Patients can also present with ophthalmoplegia and respiratory failure. Fever is usually absent. The main clinical dilemma is differentiating infantile botulism from Guillain-Barré syndrome (GBS). Deep tendon reflexes and normal cerebrospinal fluid (CSF) findings are found in infantile botulism patients, whereas elevated CSF protein levels and absent deep tendon reflexes are common in GBS. Management is primarily supportive. Monitoring, typically in an ICU setting, is required for the first week of symptoms when the risk of respiratory failure is highest. Use of antitoxin is not recommended for infantile botulism because of the belief that there are low circulating levels of toxin in this disease. Education about washing foods and objects that the infant places in his or her mouth is an important factor in prevention.
A 2-year-old child has a history of recurrent attacks of acute onset edema of tongue/throat with stridor, without urticaria. This edema happens without any precipitating factor; swelling gradually worsens over 12-36 hours and then subsides over a period of days. He has a family history of similar episodes in his father and paternal uncle. Which one of the following options is a good screening test to diagnose his condition?
a. Serum C3 level
b. Serum C4 level
c. Serum C1 level
d. Absolute eosinophil count
e. Histamine challenge test
b. Serum C4 level
Hereditary angioedema (HAE) is a rare, autosomal dominant disorder of C1 inhibitor (CIINH) deficiency. CIINH deficiency presents in adolescent or early childhood with recurrent attacks of submucosal or subcutaneous edema. The edema and swelling gradually worsens for 12-36 hours and then subsides over a period of days. Edema of the upper respiratory tract, which can variably affect some or all of the tongue, pharynx and larynx, can be life threatening. Submucosal edema of the G.I tract can present as abdominal pain, nausea and vomiting. Urticaria is not a presenting feature of CIINH deficiency. Serum C4 level has been recommended as a screening test for CIINH deficiency, as serum C4 is invariably low in untreated HAE (C4
A 3-week-old neonate with transposed great arteries and intact ventricular septal defect returns to the ICU following an arterial switch operation. HR is 185/min, BP is decreased to 45/30 mm Hg, and the left atrial pressure is elevated at 17 mm Hg. Right atrial pressure and the ECG tracing are normal. Which of the following is the most likely cause of these findings?
A. Left ventricular pump failure
B. Mitral valve insufficiency
C. Left ventricular outflow tract obstruction
D. Pulmonary hypertension
E. Pericardial tamponade
Answer: A - Left ventricular pump failure
Left ventricular pump failure is the most likely cause of low cardiac output syndrome with increased left atrial pressure in transposition of the great arteries with intact ventricular septal defect (TGA-IVS). This is because of inappropriate adaptation of the left ventricle to cope with higher resistance of systemic circulation compared to pulmonary circulation after the arterial switch operation. Mitral valve insufficiency and left ventricular outflow tract obstruction also can occur but not specifically in the case of postoperative TGA-IVS. Pulmonary hypertension rarely happens in TGA-IVS. Pericardial tamponade can occur and is common postoperative to all types of cardiac surgery.
The dicrotic notch on the aortic pressure curve is caused by which one of the following occurrences?
A - Closure of the mitral valve
B - Closure of the tricuspid valve
C - Closure of the aortic valve
D - Closure of the pulmonary valve
E - Rapid filling of the LV
C - Closure of the aortic valve
The blood forced into the aorta during systole not only move the blood in the vessels forward but also sets up a pressure wave that travels along the arteries. The pressure waves expand the arterial wall as it travels, and the expansion is palpable as the pulse. The dicrotic notch—a small oscillation on the falling phase of the pulse wave—is caused by vibrations set up when the aortic valve is snapped shut.
Ventricular afterload is best approximated by which of the following values?
A. Ventricular end-diastolic pressure
B. Ventricular end-diastolic volume
C. Systolic blood pressure
D. Mean arterial pressure
E. Ventricular wall stress
Answer E - Ventricular wall stress
The ventricular afterload is best approximated by ventricular wall stress, or the degree of stretching of the ventricular muscle. It is approximated by the formula:
Wall Stress = (P × r)/2t,
pressure (P) times the radius (r) divided by twice the wall thickness (t).
A 2-day-old neonate presents with lethargy, vomiting, and seizures. An inborn error of metabolism is suspected. The blood glucose level is normal, there is no acidosis, and the ammonia level is 950 μmol/L; urea cycle defect is diagnosed. Which of the following enzyme abnormalities is most likely involved in this patient?
A. Argininosuccinic acid lyase (ASL) deficiency
B. Argininosuccinic acid synthetase (ASS) deficiency
C. Branched-chain alpha-ketoacid dehydrogenase complex
D. Carbamyl phosphate synthetase 1 (CPS1) deficiency
E. Ornithine transcarbamylase (OTC) deficiency
Answer: E - OTC deficiency
Ammonia is normally produced from catabolism of amino acids and at high levels is a neurotoxin. In humans, ammonia is detoxified to urea in the liver by the urea cycle. Any increase in blood ammonia levels reflects an impairment of the urea cycle or extensive hepatic damage. Signs and symptoms of hyperammonemia include anorexia, irritability, lethargy, vomiting, somnolence, disorientation, and subsequently cerebral edema, coma, and death. The urea cycle defects are listed below. ASL, ASS, CPS1, and OTC are all urea cycle defects. However, OTC deficiency is by far the most commonly reported defect in the neonatal period and is often evident in the first few days of life.
An infant with OTC deficiency may be lacking in energy (lethargic) or unwilling to eat, and have poorly controlled breathing rate or body temperature. Some babies with this disorder may experience seizures or unusual body movements, or go into a coma. Complications from OTC deficiency may include developmental delay and intellectual disability. Progressive liver damage, skin lesions, and brittle hair may also be seen in any of the urea cycle defects. Children may have complete or partial expression of the gene and present later in life. Deficiency of the branched-chain alpha-ketoacid dehydrogenase complex causes maple syrup urine disease, one of the organic acidemias.
Which one of the following options describes why a higher dose of water-soluble agents are required in younger children as opposed to older children?
A - Greater rates of plasma protein binding
B - Lower rates of plasma protein binding
C - Erratic drug absorption
D - Greater proportion of body weight is water
E - Lower proportion of body weight is water
D - Greater proportion of body weight is water
This question describes why a higher dose of water-soluble agents are required in younger children as opposed to older children. Although plasma protein binding is important in drug distribution, options A and B are incorrect because they do not result in the need for a greater dose of water-soluble drugs in younger children versus older children. Option C is incorrect because water-soluble drugs are not administered in higher doses to younger children due to erratic absorption. Option D is correct while option E is incorrect because as a child ages, the percentage of their body that is water decreases, resulting in differences in drug volume distribution.
Which one of the following options is the most important action of epinephrine in terms of usage in cardiac arrest?
a. β1 causing increase in contractility
b. β2 causing vasodilation
c. α1 causing systemic vasoconstriction
d. All actions are equally important
e. PDE III inhibition inhibiting release of cAMP
c. α1 causing systemic vasoconstriction
The alpha-adrenergic mediated vasoconstriction due to epinephrine, increases aortic diastolic pressure, and thus coronary perfusion pressure—a critical determinant of successful resuscitation from cardiac arrest. At low doses, the beta-adrenergic effects may predominate, leading to decreased systemic vascular resistance; in the doses used during cardiac arrest, the vasoconstrictive alpha effects predominate.
A 5-year-old child is status-post a cardiac transplant for dilated cardiomyopathy. Which one of the following statements is true regarding his heart rate and use of inotropic medications in postoperative period?
A - Transplanted hearts have a lower basal rate due to denervation
B - Dopamine has an accentuated inotropic effect on the transplanted heart
C - Atropine is ineffective for the treatment of bradycardia in immediate post-cardiac transplant
D - Stress and exercise induced heart rate increase remains preserved
E - Permanent pacing wires are very frequently required in transplanted hearts
C - Atropine is ineffective for the treatment of bradycardia in immediate post-cardiac transplant
Loss of cardiac innervation in heart transplant patients results in significant changes in patient’s ability to respond to inotropic medications. The donor sinoatrial (SA) node—which controls the rate of the transplanted heart—does not receive vagal stimulation and thus fires at a higher basal rate. The stress and exercise induced increase in heart rate are blunted relative to normal. There is decreased cardiac response to dopamine; however, epinephrine and NE tend to have an accentuated inotropic effect on a denervated heart. The vagolytic effects of atropine are ineffective for the treatment of bradycardia. Temporary pacing wires are frequently used in the early postoperative period to maintain an adequate heart rate and cardiac output. Permanent pacing is rarely required.
You are consulting on a previously healthy, 6-month-old girl in the emergency department (ED). She has fever to 39.3 C (102.8 F), irritability, a bulging fontanelle, and what her parents describe as seizure activity at home. The ED staff have obtained a complete blood count, blood and urine culture, and a coagulation profile with the following results: WBCs, 20,500/μL; hemoglobin, 10 g/dL; hematocrit, 30%; platelets, 7,000/μL; prothrombin time, 20 seconds; international normalized ratio, 2.3; partial thromboplastin time, 60 seconds; and fibrinogen, 300 mg/dL. She has not yet received any antibiotics. You suspect meningitis and would like to perform a lumbar puncture to confirm the diagnosis. Hemodynamics are appropriate and you feel she would tolerate the procedure at this time. What would be the safest and most helpful sequence of events?
A. Administer vancomycin and cefotaxime, transfuse cryoprecipitate and platelets, perform the lumbar puncture once coagulopathy is improved, and adjust antibiotics if needed once cerebrospinal fluid (CSF) results are available.
B. Administer vancomycin and cefotaxime, transfuse fresh frozen plasma and platelets, perform the lumbar puncture once coagulopathy is improved, and adjust antibiotics if needed once CSF results are available.
C. Perform lumbar puncture while platelets are infusing, administer vancomycin and cefotaxime once lumbar puncture is complete, and adjust antibiotics if needed once CSF results are available.
D. Perform the lumbar puncture while factor VII is infusing, and administer antibiotics based upon the results of the Gram stain.
E. Transfuse fresh frozen plasma and platelets, perform the lumbar puncture, and administer antibiotics based upon the results of the Gram stain.
Answer: B
In a patient with suspected meningitis and especially sepsis, antibiotics should not be delayed while waiting to perform lumbar puncture. In this particular patient, it will be at least an hour’s delay for blood products to become available and probably at least another hour to infuse them. This time delay in receiving antibiotics is unacceptable. While giving antibiotics prior to obtaining lumbar puncture may diminish your chances of obtaining antibiotic sensitivities to the causative organism, the cerebrospinal fluid should still show biochemical changes of meningitis and can still have bacterial antigen studies performed, and the blood culture may reveal positive results.
Patients with coagulopathy are at risk for epidermal hematomas with lumbar puncture. This patient’s coagulopathy is most likely caused from disseminated intravascular coagulopathy as evidenced by a low platelet count, prolonged prothrombin time, and partial thromboplastin time. This is most quickly corrected by transfusion of platelets and fresh frozen plasma. Cryoprecipitate would not be as helpful, as the patient’s fibrinogen level is more than adequate. Cryoprecipitate is typically used to replace fibrinogen and factor VIII, while fresh frozen plasma contains multiple clotting factors and is more helpful in correcting a decrease in multiple clotting factors as is seen in disseminated intravascular coagulation. Factor VII could be used to correct the coagulopathy. However, due to the expense and high risk of complications from thromboses, it would not be the best choice. It is typically reserved for patients with life- threatening bleeding not controlled with standard blood product replacement.
- After motor vehicle accident, a 5-year-old boy is admitted to PICU for observation. On examination, he has bruises on right side of chest and diminished air entry on the right side. He had hypotension on admission which responded to fluid boluses. His oxygen saturation on room air is 94%. Chest X-ray shows good-sized pneumothorax on right side. Which one of the following treatment plans is the safest for this patient?
a. Needle decompression followed by chest tube insertion on the right side
b. Needle decompression and observation without chest tube
c. Place child on 100% oxygen and do no other intervention
d. Continue to monitor without any intervention
e. Fibro-optic bronchoscopy at bed side
a. Needle decompression followed by chest tube insertion on the right side
This is a patient with tension pneumothorax. Recommended treatment for child with tension pneumothorax is needle decompression followed by chest tube insertion. Option A is correct because this is the recommended treatment. Needle decompression will convert tension pneumothorax to simple pneumothorax and chest tube is needed regardless of the response to needle decompression. Option B is incorrect. Needle decompression alone is not sufficient because pneumothorax can recur and can lead to further complications. Options C and D are incorrect because this approach can place child at risk for further deterioration. Option E is incorrect because bronchoscopy is not indicated.
Pulmonary hypertensive crisis after surgery in the neonatal period mostly occurs in patients with which of the following cardiac lesions?
A. Truncus arteriosus
B. Moderate ventricular septal defect
C. Patent ductus arteriosus
D. Transposition of the great arteries with intact ventricular septal defect
Answer: A - Truncus arteriosus
Truncus arteriosus is the type of congenital cardiac lesion most likely to have a severe course of pulmonary hypertension after surgery because most of the blood unrestrictedly flows to the lung. Intermediate ventricular septal defect and patent ductus arteriosus(PDA) that also have high flow to the lung due to left-to-right shunt are unlikely to have pulmonary hypertensive crisis after surgery, except for some large PDAs and complete atrioventricular septal defects that have undergone surgery in older age. Transposition of the great arteries with intact ventricular septal defect is hardly seen complicated with pulmonary hypertension.
Patients with left-to-right shunt repaired at an older age are at increased risk for the complication of postoperative pulmonary hypertension, presumably due to having more time to develop significant vascular remodeling before repair.
The left ventricle pressure-volume curve at times A and B is shown in the figure. The increase in stroke volume shown at time B is due to which of the following factors?
A. Increased contractility
B. Increased preload
C. Decreased afterload
D. Improved lusitropy
Answer: D - Improved lusitropy
The left ventricle pressure-volume curves show an increase in stroke volume at time B compared with time A. The curves labeled A and B are representative of the end-diastolic pressure-volume relationship. Curve B shows improved diastolic function with improved end- diastolic volume for a similar end-diastolic pressure compared to Curve A, which results in an increase in stroke volume.
The cause of the syndrome known as thrombotic thrombocytopenic purpura is best described by which one of the following definitions?
A - Factor H deficiency, leading to constitutive activation of the alternate pathway of complement
B - Production of antibodies against platelet GP IIb/IIIa receptors, leading to antibody/platelet complexes
C - Low circulating C3 levels
D - Elevated von Willebrand cleaving protease enzyme activity level
E - Decreased von Willebrand cleaving protease enzyme activity level
E - Decreased von Willebrand cleaving protease enzyme activity level
Factor H deficiency is noted to be the cause of non-diarrhea-associated hemolytic uremic syndrome. Antibodies against GP IIb/IIIa receptors are identified as the etiology of immune thrombocytopenic purpura. Low-circulating C3 levels are seen in non-diarrhea-associated HUS, but this is not the underlying etiology for the disorder. Elevated levels of von Willebrand cleaving protease (ADAMTS-13) enzyme activity levels are seen in diarrhea-associated HUS. Decreased ADAMTS-13 levels are seen in TTP.
Aortic stenosis
A. Curve A
B. Curve B
C. Curve C
D. Curve D
E. Curve E
Answer: A
The pressure-volume loop is a useful concept for analyzing the relationship between contractility and loading conditions of the heart. In Figure B, Point D is the beginning of diastole. Point A is the end of diastole prior to ventricular contraction. From point D to point A there is diastolic filling. Point B is the end of isovolumetric contraction. At this point, the aortic valve opens. From point B to point C, there is isotonic contraction, and blood is ejected from the left ventricle. At point C, the aortic valve closes. From point C to point D, there is isovolumic relaxation.
In aortic stenosis, there is a marked increase in systolic pressure and end-diastolic pressure and marked decrease in stroke volume (curve A). The increased afterload results in elevated wall tension and decreased myocardial perfusion. A large ventricular septal defect results in shift of the pressure volume curve to the right with increased end-diastolic volume (curve C). Unlike the normal heart, increased preload will not result in increased stroke volume due to the maximal dilation of the heart.
Which of the following best characterizes the mechanisms of action of hypertonic saline solution for patients with elevated intracranial pressure?
A. Hypertonic saline solution freely crosses the blood-brain barrier and thereby reduces cerebral edema.
B. Hypertonic saline reduces the blood viscosity and free radical production in the cerebral circulation.
C. Infusion of hypertonic saline solution is used to maintain high intravascular osmolality as long as serum osmolality is maintained below 360 mOsm/L.
D. Treatment of traumatic brain injury with hypertonic saline solution causes central pontine myelinolysis.
Answer: C
The blood-brain barrier is nearly impermeable to both mannitol and hypertonic saline solution. The latter has been associated with fewer interventions to maintain an ICP of more than 15 mm Hg.
Hypertonic saline solution may be preferred over mannitol during the initial phase of management of traumatic brain injury because mannitol causes an osmotic diuresis, and low volume status can increase the risk of hypoperfusion. Maintaining intravascular volume status associated with hypertonic saline solution provides a reason why few adverse events occur when compared with mannitol and with sodium levels as high as 160 mEq/L. Mannitol is associated with transient blood viscosity. Hypertonic saline solution is administered as a bolus (10 mL/kg) and the drip is usually administered at a rate of 0.1 to 1 mL/kg/h.
Infusions can be continued if serum osmolality is below 360 mOsm/L. Use in patients with traumatic brain injury has not been associated with either central pontine myelinolysis or extrapontine myelinolysis.
Which of the following is characteristic of blood pressure after surgical repair of coarctation of the aorta?
A. Late hypertension is due to persistent hyperreninemia.
B. The incidence of postoperative hypertension is highest in children with repair early in life
C. The incidence of late hypertension is not affected by early repair
D. Early postoperative hypertension is biphasic, with the early phase primarily systolic and the late phase primarily diastolic.
Answer: D - Early postoperative hypertension is biphasic
Early postoperative hypertension is common following coarctation repair. Population-based studies indicate that the incidence of postoperative hypertension is higher in children with repair later in life. Control is important to minimize the risk of postoperative bleeding at the anastomosis site, minimize the risk of aneurysm formation in the poststenotic segment, and alleviate postcoarctectomy syndrome. The etiology is multifactorial; the pattern is typically biphasic. Early hypertension is thought to be explained by surgical stimulation of the sympathetic nerves located between the media and adventitia of the aortic isthmus. Norepinephrine released by this stimulation leads to immediate postoperative hypertension. The hypertension leads to renin release, which causes secondary hypertension. During this early phase, hypertension is primarily systolic, whereas diastolic hypertension is typically seen 2 to 3 days postoperatively. Renin levels return to normal by 7 days postoperatively; thus, hyperreninemia is not the cause of late hypertension. Balloon angioplasty does not stimulate norepinephrine or renin release, so hypertension is not seen.
A 50-kg (110-lb) child admitted to the pediatric ICU has a urine output of 75 mL/h. His serum and urine electrolyte levels are as follows: sodium, 139 mEq/L; chloride, 100 mEq/L; blood urea nitrogen, 28 g/dL; potassium, 3.5 mEq/L; bicarbonate, 24 mEq/L; creatinine, 2 mg/dL; urine sodium, 30 mEq/L; urine creatinine, 10 mg/dL. The patient’s fractional excretion of sodium is:
A. 2.3%
B. 4.3%
C. 9.3%
D. 25.3%
Answer: B - 4.3%
Fractional Excretion of Sodium (FeNA)=[(Urine Sodium × Plasma Creatinine)/(Plasma Sodium × Urine Creatinine)]100.
Urine chemistries, such as fractional excretion of sodium (FeNa) and fractional excretion of urea, are useful for differentiating prerenal acute kidney injury (AKI) from acute tubular necrosis in select patients. Evaluation of patients with AKI has become more standardized through the use of such definitions as the Risk-Injury-Failure-Loss-End Stage (RIFLE) and Acute Kidney Injury Network (AKIN) criteria to diagnose and classify this entity. These criteria, however, do not permit differentiation of the various types of AKI, including prerenal and acute tubular necrosis, which require nonhomogeneous management. FeNa is based on the premise that intact tubules will reabsorb sodium in the prerenal setting, whereas injured tubules occurring with acute tubular necrosis will not. FeNa less than 1% suggests a prerenal cause of AKI; FeNa greater than 3% suggests a renal cause of AKI.
Which one of the following options causes about 36% of pediatric patients to experience little or no analgesic effect from codeine?
A - Drug-drug interactions resulting in poor absorption
B - Polymorphism in melanocortin-1 receptor resulting in ultrarapid metabolism
C - Polymorphism in melanocortin-1 receptor resulting in poor metabolism
D - Polymorphism in CYP2D6 resulting in ultrarapid metabolism
E - Polymorphism in CYP2D6 resulting in poor metabolism
E - Polymorphism in CYP2D6 resulting in poor metabolism
This question describes one of the causes for up to 36% of pediatric patients to experience little or no analgesic effect from codeine. Option A is incorrect because significant drug-drug interactions that may cause limited effect of codeine in up to 36% of children is unlikely. Options B and C are incorrect because a polymorphism in melanocortin-1 receptor is associated with decreased sensitivity to the anesthetic desflurane, and not codeine. Option D is incorrect while option E is correct because a polymorphism in CYP2D6 results in poor metabolism of codeine into its active metabolite morphine, resulting in up to 36% of children experiencing little or no analgesic effect.
Maximum reduction in incidence of central line blood stream infections (CLABSI) in pediatric ICUs can be best accomplished by which one of the following measures?
a. Routine use antimicrobial coated central venous catheters
b. Strict implementation of catheter care bundles
c. Regular use of chlorhexidine gluconate (CHG) or silver alginate impregnated dressings
d. Regular change of catheters once every 2 weeks, even with no signs of inflammation
e. Use of chlorhexidine gluconate or povidone iodine before the insertion of central line in all children
b. Strict implementation of catheter care bundles
Option A is incorrect. Even though antimicrobial- coated (rifampin or minocycline) catheters can reduce colonization rate in adults, there is no data in children which supports the use antimicrobial-coated central line infections to prevent CLABSI. Option B is correct. To date, the most effective strategy to minimize CLABSI is to use of evidence-based central line care bundles. Option C is incorrect. CHG or silver alginate impregnated dressings alone are not proven to prevent CLABSI. Option D is incorrect. Regular change or rotation of asymptomatic catheters has not shown to be beneficial. Option E is incorrect. Chlorhexidine is better than povidone iodine in preventing CLABSI in children older than 2 months of age.
A 4-year-old is admitted to the pediatric ICU with severe traumatic brain injury. The intracranial pressure is 25 mmHg. Which one of the following values of intracranial pressure indicates the need for treatment to commence?
A - 5 mmHg
B - 10 mmHg
C - 20 mmHg
D - 30 mmHg
E - 40 mmHg
C - 20 mmHg
Poor outcomes have been associated with an intracranial pressure of higher than 20 mmHg. The normal intracranial pressure is usually less than 10 mmHg in adults, and varies between 3-7 mmHg in younger children; however, normal pressure is less than 6 mmHg in infants. The guidelines for severe, pediatric traumatic brain injury state that treatment should begin withan ICP of greater than or equal to 20 mmHg.
A 2-year-old boy is admitted to PICU immediately after accidental ingestion of his grandmother’s digoxin. Child is awake, alert, and has no symptoms. Electrolytes show sodium of 136 mEq/L, potassium of 6.9 mEq/L, chloride of 105, CO2 of 17 mEq/L, BUN of 10 mg/dL, and creatinine of 0.4 mg/dL. Which one of the following drugs is most appropriate for immediate management of this patient?
a. Intravenous furosemide 2 mg/kg
b. Intravenous glucose and insulin
c. Intravenous sodium bicarbonate 2 mEq/kg IV
d. Intravenous digoxin-specific Fab antibody fragments (DigiFab)
e. Intravenous lidocaine
d. Intravenous digoxin-specific Fab antibody fragments (DigiFab)
This is a child with digoxin toxicity. Digoxin blocks the Na+, K+-ATPase pump, leading to intracellular loss of K+ and gain of Na+ and Ca2+. Digoxin has a very narrow therapeutic index. Therapeutic plasma digoxin concentrations are 0.5-2.0 ng/mL; a level of >2 ng/mL is considered toxic and a level of >6 ng/mL is considered potentially fatal. Indications for digoxin –specific Fab antibody. fragments include life-threatening dysrhythmias, K+ value of >5-5.5 mEq/L in the setting of acute overdose, serum digoxin level of >15 ng/mL at any time, or >10 ng/mL 6 hours after ingestion, and ingestion of >4 mg in children or >10 mg in adults. Options A, B, and C are incorrect. This patient has digoxin toxicity and has potential to have fatal levels of digoxin. Treatment should focus on specific treatment rather than symptomatic treatment in this patient without any clinical symptoms other than hyperkalemia. Option D is correct response. Serum potassium levels of >5.5 is an indication for DigiFab. Option E is incorrect. Lidocaine can be used if patient has ventricular arrhythmias but this patient is asymptomatic.
In a head-injured patient, which of the following best explains the movement from Point 2 to Point 3 on the intracranial pressure curve depicted in the figure?
A. Hypothermia
B. Vasoconstriction
C. Cerebrospinal fluid drainage
D. Hypocarbia
E. Seizure
Answer: E - Seizure
The diagram is a reflection of the changes in intracranial compliance during a variety of disease states. The Monro-Kellie hypothesis states that the cranial vault contains a fixed volume consisting of 3 basic components—brain (80%), blood (10%), and cerebrospinal fluid (CSF) (10%)—which are encased by thick, inelastic dura mater and the semi-rigid cranium. These components exist in a state of volume-pressure equilibrium, and expansion of one requires a reduction in one or both of the other components to maintain a normal intracranial pressure. As the figure demonstrates, while movement from Point 1 to Point 2 (ie, by tumor, edema, or hemorrhage) is accompanied by an increase in brain volume, the intracranial pressure remains normal. This occurs by the initial displacement of CSF down the craniospinal axis followed by displacement of cerebral venous volume as well. Point 2 represents the point where even a slight increase in volume (ie, tumor edema, obstructive hydrocephalus, hemorrhage, etc) will lead to exponential rises in intracranial pressure that can be life- threatening as there is no further ability to compensate with decreased volume of the other 2 components. Point 3 represents a decompensated state and a neurosurgical emergency. A seizure will lead to hypoxia, hypercarbia, increased cerebral blood flow, increased metabolic demand, and possible ischemia, all contributing to the movement from Point 2 to Point 3.
Hypothermia, vasoconstriction, and hypocarbia would all reduce cerebral blood flow and hence improve, rather than worsen cerebral compliance.
A 19-year-old man with stem cell transplantation after recurrent treatment and relapse for acute lymphoblastic leukemia is admitted to the pediatric ICU with acute respiratory failure and altered mental status. He is now mechanically ventilated and arouses on exam to stimulation but does not communicate even with short, nonverbal response options. The patient has no written advance directive and has been estranged from his parents and has not spoken to them in over a year. He has been dating his fiancée for the past 4 years and has been living with her for the past year. Which of the following people will make decisions for the patient?
A. His fiancée, using best interest
B. His fiancée, using substituted judgment
C. His parents, using directive judgment
D. His parents, using substituted judgment
Answer: B - His fiancee, using substituted judgment
This patient currently has lack of capacity with altered mental status and inability to express his wishes for his care. Since he has no prewritten advance directive naming a surrogate decision maker or guidance for decision makers, a surrogate must be identified. The parents of a minor would be the de facto surrogates in most cases and would use best-interest criteria for decision making. In the case of a young adult (of legal consenting age), parents might use substituted judgment, but this would require a relationship in which the patient would express the wishes that the surrogate would use to make decisions. In this specific case the patient is estranged from his parents (implying an adversarial relationship) and the long-standing relationship with the woman who is now his fiancée would be likely to produce the appropriate venue for surrogate with the appropriate knowledge to apply substituted judgment. The important concepts here are the differences between substituted judgment and best interest. Directive judgment is not an ethical principle.
When using a Mapleson D system of bag and mask ventilation during sedation of an infant, which one of the following factors is the main determinant of rebreathing?
A - Length of the connecting tubing
B - Size of the breathing bag
C - Compliance of the tube
D - Size of the mask
E - Rate of fresh gas flow
E - Rate of fresh gas flow
A Mapleson D system is a system in which a tube carries fresh gas, traveling inside an outer reservoir tube, to the endotracheal tube connector. The source of fresh gas is delivered close to the mask. The control of rebreathing on such a circuit is mainly determined by the gas flow rate.
The oxygenation index (OI), one indicator of the severity of lung injury, can be calculated by the following formula:
A. (FIO2 × 100 × PaO2)/Mean Airway Pressure
B. (Mean Airway Pressure × FIO2 × 100)/PaO2
C. (Mean Airway Pressure × PaO2)/FIO2 × 100
D. PaO2 – FIO2 × 100/Mean Airway Pressure
E. PaO2/FIO2 × 100
Answer: B
That is the equation.
The oxygenation index is a calculation used in intensive care medicine to measure the fraction of inspired oxygen (FiO2) and its usage within the body.
A lower oxygenation index is better - this can be inferred by the equation itself. As the oxygenation of a person improves, they will be able to achieve a higher PaO2 at a lower FiO2. This would be reflected on the formula as a decrease in the numerator or an increase in the denominator - thus lowering the OI. Typically an OI threshold is set for when a neonate should be placed on ECMO, for example >40.
You are assessing a 16-year-old male who was intubated and mechanically ventilated at an outside hospital for respiratory failure secondary to a worsening bilateral pneumonia with a normal echocardiogram. On examination, he has poor lung compliance and is on an FIO2 of 0.35 with a PaO2 of 72 cm H2O. Which of the following best describes this patient’s acute respiratory distress syndrome stage by Berlin criteria?
A. Severe, PaO2/FIO2 (P/F) ratio less than 100
B. Moderate, P/F ratio of 100-200
C. Mild, P/F ratio of 200-300
D. Patient does not meet criteria for ARDS
Answer: C (Mild)
A PaO2/FIO2 (P/F) between 200 and 300 meets the definition of mild acute respiratory distress syndrome (ARDS) according to the Berlin consensus definition. In this case, the P/F ratio is 72/0.35 = 205. Prior to the Berlin Definition, this patient would have qualified as having acute lung injury (ALI) and not acute respiratory distress syndrome. The Berlin Definition has standardized the terms.
A 6-month-old boy with a single ventricle that was previously palliated with an aortopulmonary shunt returns from surgery after undergoing a bidirectional cavopulmonary anastomosis. His oxygen saturation is 65%. What is the most likely cause for the systemic desaturation?
A. Pulmonary arteriovenous malformations
B. Decompressing venous collaterals
C. Atrioventricular valve regurgitation
D. Increased pulmonary vascular resistance
Answer: D (Increased PVR)
Possible causes of hypoxemia following a bidirectional cavopulmonary anastomosis can be grouped into 3 categories: pulmonary venous desaturation, systemic venous desaturation, and decreased pulmonary blood flow. Decompressing venous collaterals are one cause of decreased pulmonary blood flow, along with undiagnosed contralateral superior vena cava (SVC). In bidirectional cavopulmonary circulation, any increase in pulmonary vascular resistance can open decompressing veins, resulting in a shunt of venous blood into the heart. Factors related to development of decompressing venous collaterals include bilateral SVC, elevated SVC pressure, and high early postoperative transpulmonary gradient. Vessels may be closed on the pre-Glenn cardiac catheterization, but a left SVC-to-coronary sinus collateral may develop postoperatively.
Pulmonary arteriovenous malformations typically cause hypoxemia months to years after the surgery. Atrioventricular valve regurgitation is less likely to affect systemic oxygenation. Increased pulmonary vascular resistance causes systemic desaturation.
Ventricular septal defect
A. Curve A
B. Curve B
C. Curve C
D. Curve D
E. Curve E
Answer: C
The pressure-volume loop is a useful concept for analyzing the relationship between contractility and loading conditions of the heart. In Figure B, Point D is the beginning of diastole. Point A is the end of diastole prior to ventricular contraction. From point D to point A there is diastolic filling. Point B is the end of isovolumetric contraction. At this point, the aortic valve opens. From point B to point C, there is isotonic contraction, and blood is ejected from the left ventricle. At point C, the aortic valve closes. From point C to point D, there is isovolumic relaxation.
In aortic stenosis, there is a marked increase in systolic pressure and end-diastolic pressure and marked decrease in stroke volume (curve A). The increased afterload results in elevated wall tension and decreased myocardial perfusion. A large ventricular septal defect results in shift of the pressure volume curve to the right with increased end-diastolic volume (curve C). Unlike the normal heart, increased preload will not result in increased stroke volume due to the maximal dilation of the heart.
A 12-year-old patient with Marfan’s syndrome and scoliosis is postoperative day 2 from spinal fusion. Today, his epidural was removed and he was started on a morphine PCA with continuous infusion ordered at 0.05 mg/kg/h and bolus dosing at 0.05 mg/kg/dose every 10 minutes. You are called to the floor to evaluate the patient due to poor responsiveness and find him to be extremely somnolent with HR of 72, BP of 108/68, RR of 8, and oxygen saturation of 94% on room air. He has 1-mm pupils, withdraws to noxious stimuli, and mumbles incoherently to sternal rub. Which one of the following series of actions is best in this case?
A - Disconnect PCA and administer 0.1 mg/kg (max 2 mg) naloxone and observe response, repeating every 3 minutes until breathing and alertness improve ( can administer up to max dose 10 mg). If he remains awake and appropriate for longer than 10 minutes, leave him on the floor; otherwise, PICU transfer is needed
B - Transfer to the PICU, discontinue the bolus PCA dosing, and reduce the continuous infusion to half
C - Transfer to the PICU, administer naloxone 0.1 mg/kg (max 2 mg) repeating as needed, discontinue the bolus PCA dosing, and reduce the continuous infusion to half
D - Disconnect PCA and administer 0.1 mg/kg (max 2 mg) naloxone repeating as needed, transfer to the PICU for observation
E - Disconnect PCA, transfer to the PICU for observation but avoid naloxone due to the possibility of inducing seizures
D - Disconnect PCA and administer 0.1 mg/kg (max 2 mg) naloxone repeating as needed, transfer to the PICU for observation
Most PCA overdoses result from medication errors (e.g., incorrect dilution) or mis-programming of the pump. It is therefore crucial to discontinue the PCA until the root cause can be determined. Naloxone (dosed as described) is indicated for reversal of respiratory depression. Since the effects of naloxone last only 45-60 minutes, and may be outlasted by the narcotic, PICU observation is warranted once an overdose has occurred. Naloxone administration may induce nonlethal withdrawal in chronic users, but is unlikely in this situation where narcotics have been administered for acute pain.
A 6-year-old child with spastic quadriplegia has been brought to the ED by his parents for vomiting/diarrhea for 3 days and decreased PO intake. His weight is 16 kg, while his weight in his pediatrician’s office 2 weeks ago was 18 kg. His admission serum chemistries reveal sodium of 160 mEq/lt, potassium of 3.8 mEq/lt, chloride of 124 mEq/lt, bicarbonate of 20 mEq/lt, BUN of 60 mg/dl, creatinine of 1.2 mg/dl, glucose of 108 mg/dl, and calcium of 9.0 mg/dl. Using normal values of Na of 145 mEQ/lt, K of 4.0 mEq/lt, Cl of 100 mEq/lt, bicarbonate of 24 mEq/lt, BUN of 15 mg/dl, creatinine of 0.5 mg/dl, glucose of 80 mg/dl, calcium of 10.0 mg/dl, and body water fraction of 0.6, which one of the following factors most closely approximates his free water deficit?
A - 1000 ML
B - 1500 ML
C - 2000 ML
D - 2500 ML
E - 3000 ML
A - 1000 ML
Free water deficit is calculated by the following equation: H2O deficit (in lts)= (Estimated body water fraction)× ( Current weight in kgs)× (plasma sodium/normal plasma sodium-1). H2O deficit= 0.6×16×160/145-1. H2O deficit= 9.6 × (1.103-1). Finally, H2O deficit is .993.
In patients with pericardial effusion and tamponade, pulsus paradoxus is seen in which of the following tamponade phases?
A. Phase 1 only
B. Phases 1 and 2 only
C. Phases 2 and 3 only
D. Phases 1, 2, and 3
Answer: B - Phases 1 and 2 only
In phase 1 tamponade, accumulation of pericardial fluid produces a rise in pericardial pressure which in turn increases right ventricular (RV) and left ventricular (LV) diastolic pressures. Cardiac output is not usually compromised. The inspiratory fall of arterial pressure may be more obvious than under normal conditions but does not usually reach the diagnostic criterion of pulsus paradoxus (a fall greater than 10 mm Hg). In phase 2 the elevated pericardial pressure becomes equal to RV diastolic pressure but remains lower than LV diastolic pressure. The transmural RV diastolic pressure approaches 0 mm Hg and accounts for a significant fall in cardiac output. The inspiratory fall of arterial pressure is exaggerated and pulsus paradoxus is present in most, but not all patients. In phase 3 tamponade, the pericardial and RV diastolic pressures equilibrate with LV diastolic pressure and these 3 pressures further increase together. Both the transmural RV and LV diastolic pressures become almost nil, consistent with minimal end-diastolic volumes. Cardiac output is severely compromised, and pulsus paradoxus is present in almost all cases.
Increased pericardial pressure obstructs both venous return and cardiac output. The classic presentation of acute cardiac tamponade is Beck’s triad (pulsus paradoxus, quiet precordium, and distended neck veins). Other common signs include tachypnea, tachycardia, narrow pulse pressure, and venous congestion (jugular venous distention, hepatomegaly, and/or crackles).
In a healthy 17-year-old volunteer, p-amino hippuric acid (PAH) was injected intravenously and its concentration in urine/blood and urine flow rate were calculated, which are as follows: (1) conc of PAH in urine (UPAH)= 12 mg/ml; (2) urine flow= 1 ml/min; and (3) conc of PAH in plasma (PPAH)= 0.02 mg/ml. If the hematocrit is 40%, which one of the following factors represents the renal blood flow?
a. 300 mL/min
b. 600 mL/min
c. 1100 mL/min
d. 1700 mL/min
e. 2000 mL/min
c. 1100 mL/min
Renal blood flow ( RBF) can be measured by applying the Fick principle (i.e., by measuring the amount of a given substance taken up per unit of time and dividing this value by the A-V difference for the substance across the kidney). Any excreted substance can be used if its concentration in the art and venous plasma can be measured and if it is not metabolized, stored, or produced by the kidney and does not itself affect blood flow. PAH is a substance which exhibits all such properties. Its extraction ratio is >90%, so for purpose of RBF measurements, the arterial concentration only is substituted for arteria-venous difference (since venous concentration is negligible). In fact, peripheral venous plasma can be used, as its PAH concentration is essentially identical to that in arterial plasma reaching the kidneys. In the current question: ERPF= UPAH ×V/ PPAH = 1× 12/0.02 = 600 ml/min. Average PAH extraction ratio 0.9. So actual RPF= 600/0.9= 660 mL/min. From the RPF, the renal blood flow can be calculated by dividing by 1 –Hct. Renal blood flow= RPF ×1/1-Hct. Finally 600×1/0.60 = 1100 mL.
A 4-year-old child is admitted to the PICU for dehydration and renal failure. His admission chemistry is as follows: (1) serum sodium of 140 meq/lt; (2) potassium of 4.0 meq/lt; (3) chloride of 105 meq/lt; (4) bicarbonate of 16 meq/lt; (5) serum creatinine of 1.5 mg/dl. Meanwhile, his urine sodium is 30 meq/lt and urine creatinine is 10.7 mg/dl. Which one of the following options is his fractional excretion of sodium?
a. 0.25%
b. 0.5%
c. 1.0%
d. 2.0%
e. 3.0%
e. 3.0%
Measuring urinary electrolyte may be worthwhile in determining the etiology of ARF. The fractional excretion of sodium (FeNa) is a measure of the kidneys ability to retain sodium. FENa is calculated by the following equation: FeNa = UNa ×PCr /UCr × PNa × 100. In this current question FeNa = 30 ×1.5/ 11 × 140 ×100= 4500/1540 which results in: 2.9. An FeNa of less than 1% signifies volume depletion or a “pre-renal” condition, and an FeNa of greater than 3% indicates intrinsic renal disease. In general, urinary sodium of 5-10 mEq/lt suggest substantial sodium retention in response to intravascular depletion and > 30 mEq/lt implies less avid sodium reabsorption, which occurs with intrinsic renal disease.
Which one of the following statements regarding tacrolimus use in post-transplant patients is most accurate?
a. Tacrolimus induced chronic nephropathy can be managed by dose reduction
b. Fluconazole increases tacrolimus metabolism requiring an increase in tacrolimus dose
c. Oral/nasogastric tacrolimus has less incidence of acute renal failure compared to IV tacrolimus
d. If renal dysfunction is complicating post-transplant course tacrolimus should be stopped
e. Calcium-channel antagonists should not be used to treat tacrolimus induced renal toxicity
c. Oral/nasogastric tacrolimus has less incidence of acute renal failure compared to IV tacrolimus
Calcineurin inhibitors are commonly used to treat acute rejection after organ transplantation. It is essential for intensivists to be familiar with this commonly used medication. Option A is incorrect because acute adverse effects of calcineurin inhibitors are related to dosage and blood concentration. Chronic nephropathy is largely irreversible and independent of dosage and blood concentration of tacrolimus. Option B is incorrect. Fluconazole is a known CYP3A enzyme inhibitor and thus can increase tacrolimus levels. Option C is the correct response. One of the strategies to prevent tacrolimus-induced renal dysfunction is to use oral/nasogastric protocol. Option D is incorrect because post-transplant renal dysfunction is generally managed by lowering the dose and target level with use of other immunosuppressive medications. Option E is incorrect because calcium-channel blockers may be beneficial in preventing nephrotoxicity induced by calcineurin inhibitors.
A 2-week-old male child, with history of tachypnea and poor feeding, was admitted to PICU with a diagnosis of bronchiolitis. The patient is on mechanical ventilation for respiratory distress with positive end-expiratory pressure (PEEP) of 8, FiO2 of 0.7, and pulse oximeter oxygen saturation is 88% to 95%. On examination, his heart rate is 160/minute, blood pressure is 65/35 mm kg, and capillary refill is 2 to 3 seconds. He has fixed split second sound, and no cardiac murmur. His liver is enlarged 4 cm below the costal margin. His urine output is 2 ml/kg/hr. His arterial blood gas shows pH of 7.34, PaCO2 of 40, PaO2 of 56, and base excess of negative 2. Rapid test for respiratory syncytial virus is negative. His chest X-ray is shown below. Which one of the following courses of action is the most appropriate for this child?
A - Fluid bolus followed by prostaglandin (PGE1) infusion to open ductus arteriosus
B - Start iNO at 20 parts per million and increase PEEP to 10
C - Echocardiography followed by surgery as soon as possible
D - Start continuous milrinone infusion, followed by epinephrine infusion
E - Place child on venoarterial extracorporeal membrane oxygenation (ECMO)
C - Echocardiography followed by surgery as soon as possible
This is a child with supracardiac total anomalous venous return (TAPVR). His chest X-ray has classic snowman appearance. This patient has pulmonary venous congestion with right heart failure. His hemodynamic status seems acceptable with no significant acidosis. Option A is incorrect because prostaglandin infusion will increase pulmonary blood flow and should be avoided. Option B is also incorrect because iNO can increase pulmonary blood flow and will worsen the situation. This patient may need iNO after surgery, but not indicated during preoperative period. Option C is the correct response because echocardiography is usually enough to diagnose total anomalous venous return TAPVR in this patient. TAPVR patients require surgical correction as soon as possible. Option D is incorrect because milrinone can increase pulmonary blood flow and epinephrine is not needed due to acceptable hemodynamic status. Option E is incorrect because this is a relatively stable patient, and there is no need to place this patient on ECMO before surgery. Some of the TAPVR patients may require ECMO after surgery, more so if they have associated hypoplastic left ventricle.
In a diver exposed to high barometric pressure, which one of the following is associated with the oxygen toxicity?
a. Convulsions
b. Euphoria
c. Somnolence
d. Rapture of the deep
e. High pressure nervous syndrome
a. Convulsions
The ambient pressure increases by one atmosphere for every 10 m of depth in seawater. Therefore at depth of 31 m in the ocean, a diver is exposed to a pressure of 4 atm. A diver must breathe air or other gases at increased pressure to equalize the increase in pressure on the chest wall and abdomen. However, at increased pressure, 100% oxygen causes central nervous system symptoms of oxygen toxicity. The main problems associated with oxygen toxicity include lung damage and convulsions. Since the harmful effects of breathing oxygen are proportionate to the PO2, they can be prevented by decreasing the concentration of oxygen in the gas mixture to 20% or less. If a diver breathes compressed air, the increased partial pressure of nitrogen can cause nitrogen narcosis—a condition also called as the “rapture of the deep.” At pressure of 4-5 atm, 80% nitrogen produces definite euphoria. At greater pressures, the symptoms resemble alcohol intoxication. The problem of nitrogen narcosis can be avoided by breathing mixtures of oxygen and helium, and deeper dives can be made. However, the high-pressure nervous syndrome (HPNS) develops during the dives with such mixtures. The condition is characterized by tremors, drowsiness, and a depression of the alpha activity in the EEG. Unlike nitrogen narcosis, intellectual functions are not severely affected; however, manual dexterity is impaired. The cost of HPNS is not settled, but it is worth noting that a variety of gases that are physiologically inert at atmospheric pressures are anesthetics at increased pressure. This is true of nitrogen and also of xenon, krypton, argon, neon, and helium. That anesthetic activity parallels their lipid solubility.
Which one of the following options is the osmotic activity in the isotonic saline?
a. 154 mOsm/lt
b. 77 mOsm/lt
c. 308 mOsm/lt
d. 145 mOsm/lt
e. 285 mOsm/lt
c. 308 mOsm/lt
The activity (concentration) of solute particles in a solution is inversely proportional to the activity (concentration) of water molecules in the solution. The solute activity in a solution is also called the osmotic activity and is expressed as osmoles (osm). The total osmotic activity in a solution is the sum of the individual osmotic activities of all the solute particles in the solution. For monovalent ions, the osmotic activity in milliosmoles (mOsm) per unit volume is equal to the concentration of the ions in milliequivalents (eEq) per unit volume. Thus, the osmotic activity in isotonic saline is 0.9% NaCl= 154 mEq Na+/lt + 154 mEq Cl-/lt = 154 mOsm Na+/lt + 154 mOsm Cl-/lt which result in 308 mOsm/lt.
A 4-year-old child is admitted to the PICU for dehydration and renal failure. His admission chemistry is as follows: (1) serum sodium of 140 meq/lt; (2) potassium of 4.0 meq/lt; (3) chloride of 105 meq/lt; (4) bicarbonate of 16 meq/lt; (5) serum creatinine of 1.5 mg/dl. Meanwhile, his urine sodium is 30 meq/lt and urine creatinine is 10.7 mg/dl. Which one of the following options is his fractional excretion of sodium?
A - 0.25%
B - 0.5%
C - 1.0%
D - 2.0%
E - 3.0%
E - 3.0%
Measuring urinary electrolyte may be worthwhile in determining the etiology of ARF. The fractional excretion of sodium (FeNa) is a measure of the kidneys ability to retain sodium. FENa is calculated by the following equation: FeNa = UNa ×PCr /UCr × PNa × 100. In this current question FeNa = 30 ×1.5/ 11 × 140 ×100= 4500/1540 which results in: 2.9. An FeNa of less than 1% signifies volume depletion or a “pre-renal” condition, and an FeNa of greater than 3% indicates intrinsic renal disease. In general, urinary sodium of 5-10 mEq/lt suggest substantial sodium retention in response to intravascular depletion and > 30 mEq/lt implies less avid sodium reabsorption, which occurs with intrinsic renal disease.
A 3.5-kg infant is intubated and sedated for viral pneumonia with a 3.5-mm endotracheal tube. She has had no significant changes in her ventilator support over the past 4 hours. Over the past 10 minutes, the end-tidal carbon dioxide pressure has risen from 44 to 58 mm Hg, and respiratory rate has increased from 34 to 58/min. There is no noticeable change in SpO2, heart rate, or blood pressure. Which of the following is the most likely explanation for the increase in end-tidal carbon dioxide and respiratory rate based on analysis of the airway graphics shown in the figure?
A. Flow dyssynchrony
B. Increased expiratory resistance
C. Mucus plugging
D. Tracheobronchomalacia
E. Water in the ventilatory circuit
Answer: C (Mucus plugging)
The flow-versus-time scalar demonstrates irregularities in both inhalation and exhalation. This represents an obstruction to gas flow. As the graphics denote a variable inspiratory flow pattern, the ventilator is able to vary flow as needed to accomplish the prescribed goals of volume or pressure delivery over the set inspiratory time. Of importance, the obstruction is somewhat variable from breath to breath and thus represents a dynamic process. The correct answer is mucus plugging, in which gas flow is being obstructed by secretions that move slightly between breaths. Water in the ventilator circuit would be seen as a fine-tooth variation in flow and generally appears as a superimposed “oscillatory” pattern. Tracheobronchomalacia would appear very similar to the graphics above, except each breath would be nearly identical.
A 6-month-old with hypoplastic left heart syndrome returns from OR after bidirectional cavopulmonary anastomosis procedure. Hemodynamics and oxygen saturations will be most optimized by which one of the following pulmonary/ventilator tactics?
A - Maintaining a PEEP as low as possible while still maintaining tidal volume of 9-10 cc/kg, with mild hyperventilation
B - Maintaining a PEEP of 4 cmH2O while still maintaining a tidal volume of 9-10 cc/kg, with mild hypoventilation
C - Maintaining PEEP at level needed to produce end expiratory lung volumes as close as possible to FRC, with mild hyperventilation
D - Maintaining PEEP at level needed to produce end expiratory lung volumes as close as possible to FRC, with mild hypoventilation
E - Maintaining PEEP and peak pressures as low as possible while still maintaining tidal volumes of 9-10 cc/kg
D - Maintaining PEEP at level needed to produce end expiratory lung volumes as close as possible to FRC, with mild hypoventilation
In referring to bidirectional cavopulmonary anastomosis: “To minimize superior vena cava pressure, it is desirable to minimize the use of positive pressure, including PEEP, following surgery. However, setting the ventilator to maintain PEEP at zero may result in atelectasis and an increase in pulmonary vascular resistance. As in the neonate with single ventricle physiology, favorable hemodynamics are most likely maintained by using ventilator settings that allow the end expiratory lung volume to approximate FRC, because pulmonary vascular resistance is lowest at FRC. In the patient with healthy lungs, minimal mean airway pressure and early tracheal extubation are often beneficial… A unique aspect of the physiology of the BCPA is that pulmonary blood flow is largely dependent on the resistance of 2 highly but differentially regulated vascular beds… Because Qp is dependent on venous return via the superior vena cava (largely made up of cerebral blood flow), maneuvers that limit cerebral blood flow may decrease pulmonary blood flow and exacerbate hypoxemia.” Thus, mild hypoventilation will improve O2 saturations via increasing cerebral blood flow.
The reabsorption of sodium and chloride play a major role in electrolyte and water balance regulated by the kidneys. Which one of the following parts of a nephron does not involve active transport of sodium?
a. Proximal tubules
b. Thin portion of loop of Henle
c. Thick ascending limb of loop of Henle
d. Distal convoluted tubules
e. Collecting ducts
b. Thin portion of loop of Henle
Normally about 60% of the filtered Na+ is reabsorbed in the proximal tubule, primarily by the Na+-H+ exchange. Another 30% is absorbed by the Na+-2Cl—K+ cotransporter in the thick ascending limb of loop of Henle (LOH), and about 7% is absorbed by the Na+ – Cl- co-transport in the distal collecting duct. The remaining of the filtered Na—about 3%—is absorbed via the ENaC channels in the collecting duct, and this is the portion that is regulated by aldosterone in the production of homeostatic adjustments in the Na+ balance. The thin portion of the LOH does not contribute to the active transport of Na+.
A 6-month-old patient with Fallot’s tetralogy is hospitalized beginning in mid-October for congestive heart failure. Three weeks later, he remains in the pediatric intensive care unit due to a requirement for continuous inotrope infusion and is listed for heart transplant. Which of the following next steps is most appropriate?
A - Palivizumab 15 mg/kg dosed monthly for up to 5 doses
B - Palivizumab 15 mg/kg dosed monthly for up to 3 doses
C - Intravenous immunoglobulin 10 ml/kg daily for 3 days
D - Ribavirin 15 mg/kg daily until RSV season ends
E - Ganciclovir 5 mg/kg daily for 5 months
A - Palivizumab 15 mg/kg dosed monthly for up to 5 doses
Palivizumab is the only effective form of prophylaxis for respiratory syncytial virus (RSV) and should be administered to children with congenital heart disease, chronic lung disease, or those born under 32 wk EGA. Essentially, up to 5 doses is appropriate for children under 2 years of age; meanwhile, up to 3 doses can be administered in patients born between 32-35 weeks prematurely. Prophylaxis is indicated in children during RSV season (generally November through March). Ribavirin and ganciclovir are not effective against RSV and intravenous immunoglobulins are not recommended, as they are far less specific than the monoclonal antibody in palivizumab.
Which one of the following statements is accurate regarding Pneumocystis Carini pneumonia in an HIV patient?
a. Prognosis is better in young children compared to older child and adults
b. Classified as protozoa on basis of DNA sequence analysis
c. CO2 retention is a problem from early on in the disease
d. Sensitivity of increased LDH in PCP pneumonia is reported to be between 80%-100%
e. Steroids are C/I in treatment of PCP pneumonia
d. Sensitivity of increased LDH in PCP pneumonia is reported to be between 80%-100%
PCP pneumonia is the most frequent reason for PICU admission in children with HIV. The incidence of PCP varies with the age of the child—the younger the child, the higher the incidence of PCP, the earlier the development of PCP and the poorer the prognosis. The etiologic agent is pneumocystis jiroveci (previously known as pneumocystis carinii). It is classified as a fungus on the basis of DNA sequence analysis. Pneumocystis jiroveci pneumonia is distinguished from other pulmonary processes by the severity of the hypoxemia, and marked increase in LDH levels. Early in the course, minute ventilation is maintained or increased, so severe hypoxemia in context of tachypnea and low PaO2 is frequently observed. An elevated LDH is the most consistent lab abnormality present in patients with PCP, sensitivity of an elevated LDH for PCP has been reported to be between 83%-100%.
A full-term infant born at a local hospital was found to be hypoxic. The pulse oximetry reading was 82% on room air. Administration of 100% oxygen by Oxy-Hood improved the saturations to 84%. Which one of the following tests would best and most cost-effective way to determine whether or not the etiology of this patient’s hypoxia is a result of cyanotic congenital heart disease?
A - Electrocardiogram
B - Echo
C - Hyperoxia test
D - Cardiac CT
E - Cardiac MRI
C - Hyperoxia test
The hyperoxia test is a test which is usually performed on an infant to determine whether the patient’s cyanosis is due to lung disease or a problem with cyanotic congenital heart disease. It is performed by measuring the arterial blood gas of the patient while he/she breathes room air, then remeasuring the blood gases after the patient has breathed 100% oxygen for 10 minutes. If the cause of cyanosis is due to poor oxygen saturation by the lungs, allowing the patient to breathe 100% oxygen will augment the lungs’ ability to saturate the blood with oxygen; thus, the partial pressure of oxygen in the arterial blood will rise above 150 mmHg. If the partial pressure of oxygen is less than 100 mmHg, the cyanosis is most likely due to cyanotic congenital heart disease. An electrocardiogram can be helpful in deciphering a heart disease, but is not used as an initial test to diagnose cyanotic congenital heart disease. Echocardiography would be the initial noninvasive test to diagnose the structural heart disease once the patient fails a hyperoxia test. Cardiac catheterization, cardiac CT, and cardiac MRI are all tests which are done either to confirm or to get additional information if echocardiography fails to answer all of the questions.
An 11-month-old near-drowning patient received bystander CPR and had a witnessed aspiration event. The child is now intubated and sedated due to her Glasgow Coma Scale score of 7 at the scene. Since admission she has an increasing oxygen requirement. Her core temperature is 30°C (86°F). Soon after arrival to the pediatric ICU she develops ventricular tachycardia. Which of the following steps should be taken first?
A. CPR with earliest defibrillation possible and usual antiarrhythmic medications
B. CPR only, with passive warming until core temperature above 33°C (91.4°F)
C. CPR only, with rapid rewarming of 4°C per hour
D. Extracorporeal rewarming
Answer: A - CPR with earliest defibrillation possible and usual antiarrhythmic medications
In moderate hypothermia (30°C-33.9°C [86°F-93°F]), it is recommended to perform CPR and defibrillate as soon as possible, with appropriate treatment of the possible causes of ventricular tachycardia, such as hypoxia, acidosis, pulmonary hypertension, hypothermia, and profound hypotension. If the core temperature were less than 30°C, defibrillation and medications would be less effective and should be limited; rewarming with cardiopulmonary support provided by extracorporeal membrane oxygenation may be a viable option. Rapid rewarming is not recommended for core temperatures over 30°C. Rewarming should occur at a rate of 1°C-2°C per hour. CPR with passive warming will take hours and delay a perfusing rhythm.
Compare the pressures, flows, and resistances in the pulmonary circulation with those in the systemic circulation.
Pulmonary blood flow is EQUAL to systemic blood flow (pulmonary and systemic circuits are in series).
However, pulmonary RESISTANCE and PRESSURE are much lower than systemic circulation
Osmotherapy is most effective in which one of the following forms of cerebral edema?
a. Cytotoxic
b. Vasogenic
c. Interstitial
d. Equally not effective in all forms
e. Equally effective in all forms
A - Cytotoxic
Cerebral edema is conventionally classified into vasogenic, cytotoxic, and interstitial edema based on its underlying pathogenesis. Vasogenic cerebral edema occurs due to disruption of blood brain barrier. Increased capillary permeability due to blood brain barriers disruption allows intravascular fluid and solutes to enter brain interstitial fluid, causing its expansion. Cytotoxic cerebral edema occurs due to accumulation of higher amount of intracellular salute compared to extracellular fluid, which leads to movement of water into the cells to maintain osmotic equilibrium. Diseases which impair cellular metabolism (hypoglycemia, rye syndrome) or sustained hyperosmolar state (hypernatremia, hyperglycemia) are associated with cytotoxic edema. Interstitial cerebral edema occurs when there is increased hydrostatic pressure gradient between the ventricle system and the brain interstitium, resulting in transependymal movement of CSF. Interstitial endemia is seen in obstructive and nonobstructive hydrocephalus. Consideration of pathophysiologic mechanism in cerebral edema has important therapeutic significance. Osmotic therapy is most effective in cytotoxic cerebral edema, whereas relief of CSS obstruction is necessary to treat interstitial cerebral edema. It is, however, important to realize that all 3 forms of cerebral edema can occur concurrently in the same patient.
How are FVC and FEV1 used to diagnose lung disease?
FVC and FEV1 are useful indices of lung disease
Normally, FEV1/FVC ~0.8 –> 80% of vital capacity can be expired in the first second of forced expiration. Also, in normal spirometry, you’ll see a change with a bronchodilator– greater than or equal to 12% in FEV1 or FVC and 200 ccs.
Obstructive lung disease such as Asthma
- FEV1 definitely decreases; FVC could decrease slightly, or not at all, or even increase slightly!
- Thus, FEV1/FVC DECREASES
- Cannot expire as much due to blocked airway → lower FEV1. FVC decreases too bc you can’t expire as much but it also “expands” a little because you’re leaving some air in the lung each time you expire.
Restrictive lung disease such as fibrosis
FEV1 decreases, but FVC decreases a lot more
–> Increased/normal FEV1/FVC
Which of the following tests is most useful for predicting neurologic outcome after cardiopulmonary resuscitation (CPR) in children?
A. Biomarkers, specifically neuron-specific enolase and S100b protein
B. Burst-suppression pattern on electroencephalography
C. CT of the head
D. MRI of the head
Answer: B - Burst suppression on EEG
Many resuscitation studies report end points of return of sustained circulation or survival to hospital discharge. Data about neurologic outcome and predictors of neurologic outcome after adult and pediatric cardiac arrest are limited. Information about the predictive value of available tests such as clinical neurologic examinations, neurophysiologic diagnostic studies (eg, electroencephalography [EEG] or somatosensory-evoked potentials), biomarkers, or neuroimaging is also limited.
What is known is that CT is not sensitive in detecting early neurologic injury. MRI with diffusion weighting can provide valuable information about hypoxic-ischemic injury in the subacute and recovery phases. Postarrest EEG showing a burst-suppression pattern is a sensitive and specific predictor of poor neurologic outcome. Somatosensory evoked potentials may be a highly sensitive and specific predictor of outcome in pediatric patients after cardiac arrest, but these tests are not standardized, and results are difficult to interpret.
Biomarkers may be useful to predict neurologic outcome. In an adult study, neuron-specific enolase levels greater than 33 μg/L and S100b levels greater than 0.7 μg/L were highly sensitive and specific for poor neurologic outcome (defined as death or persistent unconsciousness). These tests have not been adequately evaluated in children.
Which one of the following characteristics of an ingested drug makes it more amenable to be cleared by dialysis?
A - Drug with large volume of distribution
B - Drugs with low-volume of distribution
C - Large molecular weight
D - Highly proteins bound drugs
E - All drugs are equally dialyzable
B - Drugs with low-volume of distribution
The ability of an extracorporeal therapy to remove an intoxicant is affected by the volume of distribution of the drug. Generally, drugs with volume of distribution of
Which one of the following brain structures is most resistant to hypoxia?
A - Brainstem
B - Cortex
C - Basal ganglia
D - Thalamus
E - Inferior colliculus
A - Brainstem
Oxygen consumption by the human brain averages about 3.5 ml/100 gms of brain/minute in an adult. This figure represents approximately 20% of the total body resting oxygen consumption. The brain is extremely sensitive to hypoxia and occlusion of its blood supply produces unconsciousness in as short of a period as 10 seconds. The vegetative structures in the brainstem are more resistant to hypoxia than the cerebral cortex, and patients may suffer from an accident such as cardiac arrest and other conditions causing fairly profound hypoxia, with normal vegetative functions but serious, permanent intellectual deficiencies. The basal ganglia use oxygen at a very high rate, and symptoms of Parkinson’s disease as well as intellectual deficit can be produced by chronic hypoxia. The thalamus and the inferior colliculus are also very susceptible to hypoxic changes.
Which one of the following conditions is associated with prolongation of Q-T interval?
a. Hyperkalemia
b. Hyperthyroidism
c. Hyponatremia
d. Hypothermia
e. Hypermagnesemia
d. Hypothermia
Causes of QT prolongation are as follows:
An 18-month-old girl presented to ED with fever of 39 °C. During examination, the patient had a generalized tonic-clonic seizure which lasted for 9 minutes. A head CT demonstrated no intracranial pathology. After lumbar puncture, the patient was admitted to PICU for observation. Her cerebrospinal fluid (CSF) results showed no white cells, glucose and protein were normal, and Gram stain was negative for bacteria, but CSF quantity was not sufficient to do routine CSF culture for bacteria. In the ED, they could not draw blood for any tests. In the PICU, patient is awake, alert, and playful. She interacts well with her parents. Her temperature is 38.2 °C, and there is no external focus of infection. Which one of the following tests will be most helpful in the acute management of this patient?
A - Electroencephalogram (EEG) and MRI of brain
B - Complete blood counts and blood culture
C - Serum metabolic and electrolyte profiles with calcium, phosphorous, magnesium, and glucose
D - Urine analysis with microscopy and urine culture if necessary
E - Repeat lumbar puncture and CSF culture to rule out bacterial meningitis
D - Urine analysis with microscopy and urine culture if necessary
This is a case of a toddler with simple febrile seizures. Emphasis in simple febrile seizure is minimal intervention and the routine laboratory and neurodiagnostic tests are unnecessary. Option A is incorrect because these tests will not help in identifying cause, nor will they help in predicting the outcome in simple febrile seizures. Option B is incorrect. Child is awake, alert, and there is no definite focus of infection. In this scenario, blood work will not help in the management of this patient. Option C is also incorrect because these are unnecessary tests in simple febrile seizures. Option D is correct. Even though children with febrile seizures do not have increased risk for urinary tract infections, urine analysis may help to identify cause for fever in this child. Prevalence of UTI in a febrile children aged 2 months to 2 years without any focus on examination is 5%. Option E is incorrect. This patient is not high risk for meningitis and repeat lumbar tap is unnecessary.
The ECG tracing shown in the figure is obtained in a 3-year-old patient who was admitted with cardiac decompensation.
What is the most likely diagnosis?
A. Myocarditis
B. Ectopic atrial tachycardia
C. Wolff-Parkinson-White syndrome
D. Anomalous left coronary artery from the pulmonary artery
E. Kawasaki disease with coronary thrombosis and myocardial infarction
Answer: A - Myocarditis
The ECG tracing shown is most consistent with a diagnosis of myocarditis. The voltages are diminished, and there is flattening and inversion of the T wave. Myocardial infarction pattern also can be seen in patients with myocarditis whose ECG tracing is characterized by wide Q waves and changes in the ST segment. Anomalous left coronary artery from the pulmonary artery is characterized by Q waves in leads I and aVL. Ectopic atrial tachycardia usually is characterized by an abnormal P wave axis, although this abnormality can occur in the sinus region and thus mimic sinus tachycardia. There is no evidence for myocardial infarction as seen with Kawasaki disease with coronary aneurysm and thrombosis. ECG tracings of patients with Wolff-Parkinson-White syndrome would show ventricular preexcitation, which can be a cause of ventricular dysfunction due to unrecognized sustained tachyarrhythmias.
A 5-year-old child with MR/CP is admitted to the PICU with pneumonia. He is noted to have a serum potassium of 2 mEq/lt. He has a complex medical history and is on multiple home medications, information of which is currently unavailable. His urine potassium is 40 mEq/lt and urine chloride is 30 mEq/lt. In view of these values, which one of the following options is the most likely cause of his hypokalemia?
a. Nasogastric drainage
b. Alkalosis
c. Diarrhea
d. Inadequate intake
e. Diuretic therapy
e. Diuretic therapy
Hypokalemia is a serum potassium of 25 mEq/lt) when magnesium depletion or diuretics are responsible. Magnesium depletion impairs potassium re-absorption across the renal tubules and plays an important role in promoting and sustaining potassium depletion. Extra renal potassium loss is a major cause is diarrhea. Potassium conc in stool is 75 mEq/lt, stool volume is normally 200 ml or less, thus little potassium is lost. In diarrheal states, daily volume can be as high as 10 lt and thus severe diarrhea can result in significant potassium depletion. In diarrheal losses, urinary potassium will be low.
A 4-year-old has been in the pediatric ICU for 3 weeks and is now in the recovery phase of acute respiratory failure secondary to influenza complicating chronic lung disease. Which of the following laboratory tests is correctly matched with its representation of the onset of protein malnutrition?
A. Albumin; 3 days
B. Prealbumin; 4 days
C. Retinol binding protein; 5 days
D. Transferrin; 1 day
Answer: D (Transferrin)
The onset of protein malnutrition in the nonacute phase of critical illness may be measured by a variety of laboratory tests, each with a different half-life. Albumin has the longest half-life and low values represent malnutrition as far back as 30-60 days with a half-life of 20 days. Transferrin represents malnutrition as far back as 12-16 days with a half-life of 8 days. Retinol binding protein represents malnutrition as far back as 1-2 days with a half-life of 10 hours. Prealbumin has a half-life of 2 days and may represent malnutrition 3-4 days in the past. It must be remembered that all these measures are acute phase reactants; levels may be depressed in the acute phase of critical illness, and interpretation requires contextual data.
A 5-month-old male patient is admitted due to cough and fever with concerns for pneumonia. Shortly after admission, he develops respiratory distress and is intubated. This is the patient’s first hospitalization. He had 2 previous doctor visits due to otitis media that resolved with appropriate antibiotics and for recurrent diarrhea and thrush. Based upon the most common defect associated with this disorder, what would you expect his flow cytometry to reveal?
A - (-) T cells, (+) B cells, (-) NK cells
B - (-) T cells, (-) B cells, (+) NK cells
C - (-) T cells, (-) B cells, (-) NK cells
D - (+) T cells, (-) B cells, (+) NK cells
E - (+) T cells, (+) B cells, (+) NK cells
A - (-) T cells, (+) B cells, (-) NK cells
This case illustrates severe combined immune deficiency. The 3 major types of lymphocyte are T cells, B cells, and natural killer (NK) cells. NK cells are a part of the innate immune system and play a major role in defending the host from both tumors and virally infected cells. T cells (thymus cells) and B cells (bursa-derived cells) are the major cellular components of the adaptive immune response. T cells are involved in cell-mediated immunity whereas B cells are primarily responsible for humoral immunity (relating to antibodies). Combined immunodeficiency is a defect in T and NK cell development, thus they are negative.
Which one of the following statements is true regarding inflicted abdominal trauma in children?
A - Death rate of abdominal trauma victims of child abuse is 5%-10%
B - All children with child abuse should be screened with abdominal CT scan
C - Trauma is not a common cause of pancreatitis in children
D - Child abuse consultants often screen for occult abdominal injury using hepatic transaminases
E - Concomitant injuries are not commonly seen in children with inflicted abdominal injuries
D - Child abuse consultants often screen for occult abdominal injury using hepatic transaminases
Abdominal trauma is the second most common cause of death in children with child abuse. Even though incidence of abdominal trauma is low in child abuse, mortality of inflicted abdominal injury is around 45%. Option A is incorrect because death rate of abdomen trauma after child abuse is higher due to delayed recognition of injuries. Option B is incorrect. Routine abdominal CT scan of all child abuse children has low yield (3%) and is not recommended. Option C is incorrect because trauma is the primary cause of pancreatitis in children. Option D is correct. Alanine aminotransferase or aspartate aminotransferase levels of >80 IU/L has 77% sensitivity and 82% specificity in diagnosing occult abdominal trauma. Child abuse consultants often use screen occult abdominal injury using hepatic transaminases.
An 18-month-old girl had repair of ventricular septal defect and returned to the ICU. She is on a ventilator and receiving tidal volume of 7 ml/kg. Her peak inspiratory pressure is 21 cm of H2O; positive end-expiratory pressure (PEEP) is 5 cm of H2O; and FiO2 is 0.3. Her O2 saturation on pulse oximeter is 99%. Her chest X-ray shows that lungs are expanded to 7 ribs. Her blood pressure on admission was 84/42 mmHg. Two hours after admission, the nurse calls you to the bedside for blood pressures readings of 60/31 mmHg. Patient’s central venous pressure reading from right subclavian line is 4 cm of H2O. Which one of the following interventions has the best chance to restore this patient’s blood pressure back to normal?
a. Five percent albumin bolus of 20 ml/kg
b. Decrease PEEP to 3 cm of H2O
c. Decrease tidal volume to 6 ml/kg
d. iNO 20 ppm
e. Milrinone infusion at 0.5 mcg/kg/minute
a. Five percent albumin bolus of 20 ml/kg
This patient’s blood pressure is low because of low right atrial pressure. Within normal physiological boundaries, right atrial pressure is the main factor that determines right ventricular output and in turn total cardiac output. Option A is correct because fluid bolus will increase right atrial pressure, right ventricular output, and cardiac output restoring blood pressure. Option B is incorrect. Lungs are not over-expanded on chest X-ray and decreasing PEEP in will not decrease pulmonary vascular resistance and will not improve blood pressure. Option C is incorrect. Tidal volumes within normal limits will not affect pulmonary vascular resistance or right ventricular output. Option D is incorrect because this patient does not have clinical features of pulmonary hypertension and iNO will not be useful for this patient. Option E is incorrect because in the setting of low venous return to right heart, milrinone can further decrease blood pressure. Milrinone is mainly used in the setting of low cardiac output state secondary to myocardial dysfunction after cardiac surgeries.
A 12-year-old boy is admitted to the ICU after he was found unconscious in a house fire. He is now intubated and on FiO2 of 0.6. His pulse oximeter reads the arterial oxygen saturations (SaO2) as 100%. His carboxyhemoglobin level is 25%. Which one of the following options best describes this patient’s SaO2 and oxyhemoglobin (oxy-Hb) dissociation curve shift?
A - SaO2 is falsely elevated and the oxygen hemoglobin dissociation curve is shifted to the right
B - SaO2 is falsely elevated and the oxy-Hb dissociation curve is shifted to the left
C - SaO2 is appropriately elevated and oxy-Hb dissociation curve is shifted to the right
D - SaO2 is appropriately elevated and oxy-Hb dissociation curve is shifted to the left
E - FiO2 of 0.6 is causing elevated SaO2 and there is no shift in the oxy-Hb dissociation curve
B - SaO2 is falsely elevated and the oxy-Hb dissociation curve is shifted to the left
Carbon monoxide has a higher affinity to bind with Hb as compared to oxygen. Option B is correct because binding of carbon monoxide to hemoglobin causes increased affinity of oxygen to bind to the remaining sites. This is reflected as a leftward shift of the oxy-Hb dissociation curve. Carboxyhemoglobin is interpreted as oxyhemoglobin by the pulse oximeter—hence, in this clinical scenario, SaO2 is falsely elevated. Options A and C are incorrect because the oxy-Hb curve is shifted to the left. Option D is incorrect as the SaO2 are falsely elevated, and cooximetry is imperative in clinical suspicion of carbon monoxide poisoning. Option E is incorrect because there is a leftward shift of oxy-Hb curve in carbon monoxide poisoning.
A severely hypoxic patient has a baseline PaO2 of 50 mmHg and PaCO2 of 80 mm Hg. An air bubble in the atrial blood gas sample would affect the results in which way?
A. The PaO2 would be decreased and the PaCO2 would be decreased.
B. The PaO2 would be decreased and the PaCO2 would be increased.
C. The PaO2 would be increased and the PaCO2 would be decreased
D. The PaO2 would be increased and the PaCO2 would be increased.
Answer: C
An air bubble consists of atmospheric gas with a partial pressure of oxygen theoretically PaO2 = 154 mm Hg and a PaCO2 = 0. Hence, this patient’s blood gas would be altered with dilution of the PaCO2 and spurious elevation of the PaO2. If the patient were not hypoxic, the PaO2 could be decreased if their PaO2 were greater than 154 mm Hg of atmospheric PO2; however, this patient is severely hypoxic with a low PaO2.
Diastolic runoff or low diastolic pressure as one pathophysiology for developing necrotizing enterocolitis occurs in which of the following cardiac congenital lesions?
A. Critical coarctation
B. Large ventricular septal defect
C. Tetralogy of Fallot
D. Truncus arteriosus
Answer: D - Truncus arteriosus
The pathophysiology of necrotizing enterocolitis (NEC) in cardiac infant patients presumably entails intestinal hypoperfusion. Diastolic runoff and low diastolic pressures in truncus arteriosus may contribute to intestinal hypoperfusion. NEC also can occur in critical coarctation and hypoplastic left heart syndrome because of critical flow of systemic circulation through patent ductus arteriosus manifested as circulatory shock. Splanchnic complications can occur with large ventricular septal defect if low cardiac output occurs caused by lung overflow and pulmonary hypertension.
Microbiology laboratory techniques that are useful in determining the correct type and dosage of an antimicrobial agent against a specific bacterial isolate include:
A. Blood culture
B. Enzyme-linked immunosorbent assay
C. Gram stain
D. Latex agglutination
E. Susceptibility tests
Answer: E - Susceptibility tests
Antimicrobial susceptibility is a commonly used method for determining the antimicrobial activity of a bacterial isolate. Antimicrobial susceptibility is expressed in terms of the minimum inhibitory concentration (MIC) and minimum bactericidal concentration (MBC) of an antimicrobial agent. The MIC and MBC indicate the degree to which a microorganism is susceptible to an antimicrobial agent, and thus are useful in determining the type and dosage of antibiotic to use. Latex agglutination and ELISA are used primarily to detect bacterial antigen in body fluids. The Gram stain may demonstrate the presence and morphologic characteristics of a bacteria, but are nonspecific. The blood culture may assist in identifying a bacterial isolate but does not assist in determining the susceptibility profile.
Which one of the following congenital cardiac abnormalities is least likely to be associated with coarctation of aorta (CoA)?
A - Supravalvular mitral stenosis
B - Parachute mitral valve
C - Subaortic stenosis
D - Transposition of great arteries
E - Pulmonary stenosis
E - Pulmonary stenosis
CoA may be a part of complex left-sided obstructions identified as Shone’s syndrome which consists of coarctation, supravalvular mitral stenosis, parachute mitral valve, and subaortic stenosis. Syndromes that have low pulmonary blood flow, such as tetralogy of Fallot, tricuspid atresia, pulmonary stenosis, and atresia are rarely associated with CoA.
A child is admitted to ICU for metabolic acidosis of unknown etiology. His measured serum osmolality is 340 mOsm/lt . His electrolytes are as follows: (1) sodium of 145 mEq/lt; (2) potassium of 3.5 mEq/lt; (3) chloride of 111 mEq/lt; (4) bicarbonate of 14 mEq/lt; (5) BUN of 28 mg/dl; (6) creatinine of 1.2 mg/dl;
a. 5
b. 10
c. 20
d. 30
e. 40
d. 30
Plasma osmolality is calculated by the following equation: 2× plasma Na + glucose/18 + BUN/ 2.8. Therefore, 2×145+ 180/18 + 28/2.8 is the equation in this question. 290 +10+10=310 mOsm/Kg H2O. Thus, the osmolar gap 30 mOsm/Kg H2O. Due to the fact that solutes other than sodium, chloride, glucose, and urea are present in the extracellular fluid, the measured plasma osmolality will be greater than the calculated plasma osmolality. This osmolar gap is normally up to 10 mOsm/Kg H2O. An increase in the osmolar gap occurs when certain toxins (e.g., ethanol, methanol, ethylene glycol, or the unidentified toxins that accumulate in renal failure) are in the extra cellular fluid.
Which one of the following options is a risk factor for propofol infusion syndrome?
a. Old age
b. Bacterial infection/meningitis
c. High carbohydrate intake
d. Low fat intake
e. Concomitant steroid use
e. Concomitant steroid use
Propofol infusion syndrome associated with a high dose (>4 mg/kg/hour) and prolonged use (>48 hours). Risk factors are as follows: (1) young age; (2) critical illness; (3) high fat and low carb diet; (4) inborn errors of mitochondrial fatty acid oxidation; and (5) Concomitant catecholamine infusion or steroid therapy. Finally signs and symptoms include severe metabolic acidosis, CV collapse, rhabdomyolysis, hyperlipidemia, renal failure, and hepatomegaly.
Which one of the following options is the correct order of organs in respect to their oxygen consumption?
a. Brain > heart > kidneys > liver > skeletal muscle
b. Heart muscle > brain > kidneys > liver > skeletal muscle
c. Heart muscle > kidneys > brain > liver > skeletal muscle
d. Skeletal muscle > liver > heart > brain > kidneys
e. Liver > brain > heart > skeletal muscle > kidneys
c. Heart muscle > kidneys > brain > liver > skeletal muscle
Resting oxygen consumption of various organs in a 63 kg adult male with MAP of 90 mmHg and oxygen consumption of 250 mL/min is seen in the following figure:
Which one of the following options is the correct order of organs in respect to their oxygen consumption?
a. Brain > heart > kidneys > liver > skeletal muscle
b. Heart muscle > brain > kidneys > liver > skeletal muscle
c. Heart muscle > kidneys > brain > liver > skeletal muscle
d. Skeletal muscle > liver > heart > brain > kidneys
e. Liver > brain > heart > skeletal muscle > kidneys
c. Heart muscle > kidneys > brain > liver > skeletal muscle
Resting oxygen consumption of various organs in a 63 kg adult male with MAP of 90 mmHg and oxygen consumption of 250 mL/min is seen in the following figure: